2015 Exam MCQ

¡Supera tus tareas y exámenes ahora con Quizwiz!

answer:B

40. What would you see in a colleague who has opioid addiction? a) Immaculate charting b) Difficult to find when on call c) Wants to work with residents d) Prefers not to provide break relief

Consensus - B Answer B - Overinflate the tracheostomy tube cuff • The first step in post tracheostomy hemorrhage is to overinflate the tracheostomy tube cuff in order to tamponade the hemorrhage • The following is from Miller describing post tracheostomy hemorrhage and management Box66-14 - Overinflate the tracheostomy cuff to tamponade the hemorrhage If this fails: - replace the trach tube with an oral ETT. Position cuff with fiberoptic bronch to just above carina - apply digital pressure to innominate If this fails: - slowly withdraw the ETT and overinflated the cuff to tamponade - proceed with definitive therapy (sternotomy and ligation of the innominate) Post tracheostomy hemorrhage • Usually from local vessels in the incision such as anterior jugular or inferior thyroid veins • Massive hemorrhage 1-6 weeks postoperative is most commonly caused by tracheoinominate artery fistula • Small sentinel bleed occurs in most patients before a massive bleed • o First branch of aortic arch divides into right carotid and right subclavian artery Miller 8th Chapter 85 pg 2542 • Postoperative problems following tracheostomy o Immediately after the procedure ♣ Bleeding, pneumothorax, subcutaneous or mediastinal emphysema, and hypoventilation or airway obstruction o Late complications ♣ Tracheal stenosis, tracheoesophageal fistula formation, tracheomalacia and tracheal necrosis • Bleeding is usually inconsequential bleeding into airway may cause patient to cough and buck forcefully • Major bleeding from large artery or vein (often communicating branch of superior thyroid artery may necessitate immediate exploration of the surgical field, whereas bleeding form the innominate artery may occur from erosion by the distal end of the cannula • Treatment includes inflating the tube cuff and pulling the tube assembly anteriorly to tamponade the bleeding • An oral tracheal tube should then be inserted for management in the operating room Regarding A • Definitive treatment Regarding C • Second step if first fails Regard D • Third step

1. We ask your help at the intensive care unit for a patient with a tracheostomy performed twelve days ago. He just had a bleeding of 300 ml from the tracheostomy. What should you do? a) Consult the thoracic surgeon to perform an emergency sternotomy with ligation of the brachiocephalic artery b) Overinflate the tracheostomy tube cuff c) Put an armed ETT just proximal to the carina d) Put an armed ETT just below the site of the tracheostomy

Answer: D Chestnut Ch. 2 Pg. 20-23 Decreased FRC - TRUE: The FRC begins to decrease in the 5th month of pregnancy, dropping by 400-700mL, to reach 80% of its pre-pregnancy value at term (ie total reduction of 20%). Elevation of the diaphragm contributes to this, with ERV reduced by 25%, and RV is reduced by 15%. Placing a term patient supine can further decrease the FRC to 70% of pre-pregnancy values, and placing the supine patient in a 30° head up position can improve supine FRC by 10%. Increased oxygen consumption - TRUE: As pregnancy progresses, oxygen consumption increases, however cardiac output increases to a lesser extent, resulting in a reduced mixed venous oxygen content. Increased cardiac output - TRUE: Cardiac output begins to increase by 5 weeks gestation, and is 35-40% above baseline by the end of the first trimester. It continues to increase throughout the second trimester until it is roughly 50% greater than pre-pregnancy values, at which it remains throughout the 3rd trimester. Cardiac output increases initially due to a rise in heart rate, occurring by the 4th to 5th week of pregnancy, and is 15-25% above baseline by the end of the first trimester. An increased stroke volume (by 20% at the end of the first trimester, and 25-30% by the end of the second trimester) accounts for the continued rise in cardiac output during the second trimester. Decreased oxygen carrying capacity - FALSE?: Maternal plasma volume begins to expand by 6 weeks gestation, and peaks at 34 weeks gestation, at an increase of approximately 50% above baseline. After 34 weeks, the plasma volume stabilizes or decreases slightly. Red blood cell volume decreases during the first 8 weeks of pregnancy, reaches the pre pregnancy level by 16 weeks, and rises a further 30% above pre-pregnancy levels at term. As a result, there is a physiologic anemia of pregnancy, but a net increase in red cell mass, and increased cardiac output and as a result an increase in oxygen carrying capacity. Strictly using the formula for oxygen carrying capacity DO2 = CO x [(SaO2 x [Hb] x 1.34) + (0.003xPaO2)], (looking at concentration of hemoglobin, instead of absolute amount), however, you do have a physiologic anemia, and as a result, you would calculate a decreased oxygen carrying capacity. Increased A-a gradient - FALSE: A gradient exists between end-tidal CO2 tension, and PaCO2 in non-pregnant women. The two measurements are equivalent during early pregnancy, at term gestation, and in the postpartum period, owing to the reduced alveolar dead space resulting from an increase in cardiac output during pregnancy. Based on this statement, presumably the reduction in dead space would result in a reduced A-a gradient, though I cannot find a statement directly to this effect. Chestnut pg 20: "Moving a pregnant woman from the supine to the erect or lateral decubitus position improves arterial oxygenation and reduces the alveolar-to-arterial oxygen gradient"

101. Parturients desaturate faster than non-parturients. All of the following contribute to this phenomenon except a) Decreased FRC b) Increased oxygen consumption c) Increased cardiac output d) Decreased oxygen-carrying capacity e) Increased A-a gradient.

Clear Liquids 2h; breast mild 4h, infant formula 6h; nonhuman milk 6h; heavy meal 8h

10. How many hours do we have to stop maternal milk before surgery? a) 2hours b) 4h c) 6h d) 8h

Answer: Decreased incidence of DVT Miller Ch. 80 Pg. 2419, Ch. 56 Pg. 1691, 1715 Increased blood loss - FALSE: The hemodynamic effects of regional anesthesia may be associated with decreased blood loss in pelvic and lower extremity surgery. Decreased incidence of DVT - TRUE: Regional anesthesia prevents postoperative inhibition of fibrinolysis, and may decrease the incidence of DVT after total hip arthroplasty. This statement is controversial, as the same benefits have not been found following total knee arthroplasty Increased incidence of wound infection - FALSE: Couldn't find a reference to wound infection risk with spinal. Decreased incidence of urinary retention - FALSE: In orthopaedic patients undergoing hip replacement, bladder catheterization was no more frequent following spinal or epidural anesthesia than it was after a general anesthetic with opioid analgesia. Having said that, in the same chapter, Miller states that male gender and advancing age are inconsistently liked with urinary retention after neuraxial anesthesia, with the administration of intrathecal morphine strongly associated with this complication. Either way, a spinal will either increase or have no impact on the incidence of urinary retention.

100. What is true regarding spinal anesthesia vs general anesthesia for hip fracture in the elderly? a) Increased blood loss b) Decreased incidence of DVT c) Increased incidence of wound infection d) Decreased incidence of urinary retention

Answer: B Chestnut Ch. 42, Pg. 743-744 Lumbosacral plexus - TRUE: Can be caused by compression of the lumbosacral trunk by the fetal head at the pelvic brim, usually in the context of cephalopelvic disproportion with prolonged labour and difficult vaginal delivery. This preferentially affects the medial fibers, which make up the peroneal nerve (not the tibial). Results in weakness in ankle dorsiflexion (foot drop), as well as a sensory disturbance of the L5 dermatome, which covers the lateral aspect of the lower leg, and the dorsum of the foot. Common peroneal nerve - TRUE: The deficits would fit (and the explanation for the lumbosacral plexus deficits specifically refer to the medial fibers of the plexus, which make up the peroneal nerve). Mechanism of injury: prolonged legs up in stirrups. Sciatic nerve - FALSE: A sciatic nerve palsy usually arises from compression of the nerve in the buttock, however it is not generally recognized as a complication of childbirth (potentially because it is mistaken for an injury to the lumbosacral trunk). Symptoms include a loss of sensation below the knee, sparing the medial side, and loss of movement below the knee (the injury in the question involves only loss of dorsiflexion, and sensory loss to the lateral lower leg and dorsum of the foot). Posterior cutaneous nerve and gluteal nerve function are preserved, suggesting the injury is distal to the lumbosacral plexus (the gluteal nerves branch off the sciatic nerve, so if the injury was at the level of the plexus, you would expect involvement of these nerves). In this case, the sparing of the gluteal and posterior cutaneous nerves suggests the injury was distal to where they branch off. Injury of the sciatic nerve is possible during childbirth under neuraxial blockade, either due to sitting in one position for a long time, or from a misplaced hip wedge during a cesarean section. Hypotension may contribute to the injury. The neuraxial technique is blamed for the injury because it either masks the discomfort which would have prompted a change in position, or the symptoms occur, but are attributed to the numbness caused by the nerve block, and are overlooked/ignored. Obturator nerve - FALSE: The obturator nerve is susceptible to injury where it crosses the pelvic brim, or within the obturator canal. Symptoms include weakness of hip adduction and internal rotation, as well as sensory deficits over the upper inner thigh. Injury can be seen after both labour and cesarean delivery. Lateral femoral cutaneous nerve - FALSE: LFCN neuropathy is commonly referred to as meralgia paresthetica. The LFCN is a purely sensory nerve, and can be entrapped as it passes around the anterior superior iliac spine beneath or through the inguinal ligament. Can be seen during pregnancy (typical onset is at 30 weeks gestation), as well as intrapartum due to increased intra-abdominal pressure. A large intraabdominal mass, or use of retractors during pelvic surgery can also cause injury. Symptoms include numbness, tingling, burning or paresthesias over the anterolateral aspect of the thigh. The symptoms usually resolve after childbirth, or may be transiently relieved by local infiltration.

102. A postpartum woman is unable to dorsiflex and has loss of sensation over her lateral lower leg and dorsum of the foot. Which is the most likely site of injury? a) lumbrosacral plexus b) common peroneal nerve (possible) c) sciatic nerve d) obturator nerve e) lateral femoral cutaneous nerve

Answer:b (should read left lateral decubitus) 1) prevent further air entry (notify surgeon - flood or pack surgical field; jugular compression; lower the head) 2) Treat the intravascular air: aspirate right heart catheter, discontinue N2O, 100% O2, pressers, inotropes, chest compression

11. All are managements of acute air embolism during neurosurgical case, except: a) Flood surgical site b) Right lateral decubitus c) Jugular compression d) Aspirate central venous catheter

Answer: D - Temperature

110. All except which of the following are part of the PACU aldrete patient rating score a) Activity b) Circulation c) Resp d) Temp

Answer:D

126. Associated with multiple gestation except a) Abruption b) Preeclampsia c) DIC d) Resp Failure

answer:A

127. Which complies with fasting guidelines a) Toast and juice 6 hours ago b) Coffee with cream 3 hours ago c) Methadone with 100 ml juice 1 hour ago d) ??

answer:A

135. Co2 absorbents contain various hydroxides, all except: a) Mg b) Na c) K d) Ca

ANSWER: B (Consensus B) A lot of research for proofs for what should have been straightforward...stupid textbooks... Regarding A: Right to left shunt would SLOW inhalational induction References: Barash Ch 17 Pg 455, Miller Ch 26 Pg 647 (Barash) The rate of Pa (arterial) relative to PI (inspired) is significantly reduced. There is less total anesthetic uptake, so the rate of rise of PA (alveolar) relative to PI increases even though induction of anesthesia is slowed because CNS partial pressure equilibrates with Pa. The rate of rise of Pa/PI is not as depressed as that for less soluble anesthetics. The greatest effect of shunting is found with the least soluble anesthetics. (Miller) Right-to-left shunting results in a difference b/w Palv and the partial pressure of anesthetic in arterial blood (Part). This is because arterial blood represents a mixture of shunted mixed venous blood with blood that equilibrates with alveolar gases. Such shunts also reduce transcapillary gas exchange in the lung and slow anesthetic uptake. Shunt reduces the ratio of Part:Palv more for insoluble anesthetics, such as N2O. Regarding B: Surprisingly (and frustratingly) there is little in Barash/Miller/Pharm + Phys regarding the effects of a shunt on IV induction. Although, intuitively, this seems a pretty straightforward concept!!! In Coexisting, in the discussion of management of anesthesia of TGA lays it out simply. This is a pretty major shunting situation. Drugs administered intravenously are distributed with minimal dilution to organs such as the heart and brain. Conversely, the onset of anesthesia produced by inhaled drugs is delayed because only small amounts of the inhaled drug reach the systemic circulation. Regarding C: (Barash Ch 34 Pg 926) Spinal and epidural blocks to midthoracic levels have little effect on pulmonary function in patients without pre-existing lung disease. Drugs used perioperatively for sedation during spinal or epidural block likely have a larger impact on pulmonary function than the block per se. In particular, lung volumes, resting minute ventilation, dead space, arterial blood gas tensions, and shunt fraction show little or no change during spinal or epidural anesthesia. Regarding D: PE would increase a (presumably intra-cardiac) right-to-left shunt by increasing PVR. (Cote Ch 15, Pg 329) In children with dependent right-to-left shunts, the shunt increases when SVR decreases or PVR increases. (Miller Ch 94 Pg 2805) Right to left shunts occur when pulmonary vascular or RVOT resistance exceeds SVR, thereby reducing PBF. The systemic circulation receives an admixture of deoxygenated blood via the shunt and manifests clinically as cyanosis and hypoxemia.

132. Right to left shunt would a) Speed inhalational induction b) Speed IV induction c) Decreased shunt with epidural use d) Decreased shunt with PE

Answer: Cote 673 Re A historically, the causative organism was typically Hae- mophilus influenzae type B.2,238-240 However, with the widespread use of H. influenzae vaccination, the incidence of epiglottitis in children has all but disappeared in medically advantaged countries.241,242 Streptococcus,243 Staphylococcus,244 Candida,245 and other fungal pathogens246 have become more frequent causes of this now rare disorder in children, although the incidence in adults has not diminished substantially.247 It should be noted that vaccine failures may occur or parents may refuse proper immunizations of their child, resulting in susceptibility to H. influenzae type B infection.248,249 Epiglottitis has become more of a disease of adults.250 Re B - kinda true? The onset is usually abrupt, with a brief history of high fever, severe sore throat, and difficulty in swallowing. Re C (false) The safest, most conservative approach to the management of acute epiglottitis is to establish an artificial airway as soon as the diagnosis is made, and then, with the airway secured, to proceed with appropriate antibiotic and supportive therapy Re D (false) Acute epiglottitis is a clinical diagnosis. It must remain prominent in the differential diagnosis of a child presenting with signs and symptoms of UAO. However, in some early cases, the clinical presentation alone may be inconclusive. If so, a lateral radio- graph of the neck will usually demonstrate a swollen epiglottis and aryepiglottic folds (thumb print) Re E and F (false) These children should not be premedicated. Instead they should be brought to the OR calm and undisturbed. If it takes the presence of a parent to achieve this, then that is what should be done. In the OR, with equipment and personnel who can insert a surgical airway immediately present, a precordial stethoscope, pulse oximeter, and other standard monitors are applied. General anesthesia is induced with oxygen and sevoflu- rane with the child still sitting up. Spontaneous respiration is continued as the child is gently allowed to recline. If the child is moribund, then an awake intubation should be considered. When a surgical stage of anesthesia is achieved, IV access is established and secured. A large fluid bolus of balanced salt solution is infused (20 to 30 mL/kg) because these children are often dehydrated and will require a deep plane of anesthesia to permit tracheal intubation while spontaneous respirations are preserved. Some anesthesiologists would also administer an anti- sialagogue to reduce secretions. Epiglottitis is marked by progres- sive swelling of the lingual surface of the epiglottis with resultant obliteration of the vallecula (see Fig. 31-15). Viewing the glottic opening without traumatizing the epiglottis may usually be accomplished by forcing the tip of the laryngoscope blade along the center of the base of the tongue into the vallecula, where the vallecula has been obliterated by the swollen lingual surface of the epiglottis (see Fig. 31-15, C). Lifting the base of the tongue, without directly touching the epiglottis, can then expose the glottis. A stylet within the ETT may be helpful because it provides increased rigidity to facilitate introduction through a partially obstructed glottic aperture. The size of the ETT should be one-half size smaller (0.5 mm ID) than otherwise selected for the same age child. By choosing an ETT with a smaller ID than usual, one also lessens the risks of pressure necrosis on the mucosa.

19. What is True of Epiglottitis? a) Hemopholus Influenza is quite common. b) Often preceded by URTI c) It is a surgical emergency d) Must get imaging for diagnosis e) Use a larger ETT f) Always start an IV before anesthetic induction

Consensus D Answer D Desflurane • Barash o Inhaled anesthetics, including N2O produce a dose dependent attenuation of the ventilator response to hypoxia o This action appears to depend on peripheral chemoreceptors o Sub anesthetic concentrations of volatile anesthetics (0.1 MAC) elicit anywhere from a 25% to 75% depression of the ventilator drive to hypoxia. (Barash page 468) • Miller o Depression of hypoxic ventilatory response ♣ Halothane > enflurane > sevoflurane = isoflurane > desflurane proposed by pandit (Miller Chapter 27 pg 693)

25. Which inhaled anesthetic blunts the response to hypoxia the least at 0.1 MAC? a) Isoflurane b) Sevoflurane c) Halothane d) Desflurane

Consensus B - 8 schools C - 5 schools A - 1 school Answer B TKA • Barash lists a table indicating 41-85% total prevalence for TKA based on use of mandatory venography in prospective randomized clinical trials in which patients received either prophylaxis or a placebo • Without prophylaxis, venous thrombosis develops in 40% to 80% of orthopedic patients, and 1% to 28% show clinical or laboratory evidence of pulmonary embolism. (Barash pg 1455) General risk factors for thromboembolism • Miller did not differentiate between which surgery was a higher risk factor for DVT just a blanket statement involving orthopedic surgeries a. Thromboembolic complications remain one of the leading causes of morbidity and mortality after orthopedic surgery. THA, total knee arthroplasty (TKA), and hip and pelvic fracture surgery have the highest incidence of venous thromboembolism including DVT and PE. (Miller 2394) Regarding C Hip fracture • Would seem to be the more obvious choice at first glance given increased amount of time for immobility post op however I could not find evidence/numbers for this in Barash/Miller. a. They do mention that "several studies have reported improved outcome with regional anesthesia compared with GA in theses patients. Patients undergoing hip fracture surgery have the greatest risk for death from PE" which may have lead some schools to pick C (Miller pg 2397)

23. Which of the following has the greatest risk of DVT? a) THA b) TKA c) Hip/femur fracture d) Tibial fracture

Answer A Hypotension • The most common cause of oliguria in the immediate postoperative period is depletion of intravascular fluid volume (Miller Chapter 96 pg 2938) • In surgical patients, ATN is the most common cause of AKI. a. ATN remains the most common ischemic lesion and represents an extension of prerenal azotemia, whereas cortical necrosis may follow a massive renovascular insult such as prolonged suprarenal aortic clamping or renal artery embolism (Barash pg 1408)

20. What is the most likely cause of perioperative renal failure? a) Hypotension b) Nephrotoxins c) Sepsis d) Thromboemboli

Consensus C Answer C • Recall that dibucaine inhibits normal butyrylcholinesterase/pseudocholinesterase to a far greater extent than the abnormal enzyme • This lead to establishment of dibucaine number a. Dibucaine inhibits normal enzyme approximately 80%, heterozygote 60%, homozygote 20% (I would memorize the dibucaine numbers in Miller with the succinylcholine apnea or Barash)

24. After a dose of succinylcholine return of ventilation does not occur for 20min after the dose given. What is the most likely dibucaine number? a) 20 b) 40 c) 60 d) 80

Consensus B B - 7 Schools C - 7 schools Answer B - The obturator nerve is located between the adductor brevis and adductor magnus - True • The anterior branch of the obturator nerve usually lies on the lateral edge between the adductor longus and brevis muscles, while the posterior branch of the obturator nerve lies between the adductor brevis and magnus muscles; however, the obturator nerve shows considerable degree of variability at this level (Barash 1008) More detailed information on the obturator nerve • Obturator nerve (L2-L4) • The obturator nerve emerges from the medial border of the psoas muscle at the pelvic brim to pass behind the common iliac vessels and lateral to the internal iliac. It then courses inferiorly and anteriorly along the lateral wall of the pelvic cavity on the obturator internus muscle towards the obturator canal through which it enters the upper part of the medial aspect of the thigh above and anterior to the obturator vessels. The nerve divides into its anterior and posterior branches near the obturator foramen. The anterior branch passes into the thigh anterior to the obturator externus, descends in front of the adductor brevis and behind the pectineus and adductor longus, with its terminal cutaneous branches emerging as it courses alongside the femoral artery. It supplies the adductor longus, gracilis, adductor brevis (usually) and pectineus (often). Cutaneous branches supply the skin on the medial aspect of the thigh and perhaps the medial knee. The nerves's posterior branch pierces the obturator externus muscle anteriorly and supplies it, then passes behind the adductor brevis muscle (sometimes supplies it) to descent to the anterior aspect of the adductor magnus (medial to the anterior branch), which it supplies. There is no apparent cutaneous supply form this nerve. It then transverses the adductor canal with the femoral artery and vein to enter popliteal fossa where it terminates as the articular branch to the knee joint capsule (oblique popliteal ligament) (Barash pg 954) Answer C - Femoral artery is important in locating the saphenous nerve - True • The proximally located larger femoral artery (rather than the more distal saphenous branch of the descending genicular artery) as a highly visible landmark seems to help identify the Sartorius muscle and nerve (Barash pg 1008) More detailed information on the femoral nerve • Femoral nerve (L2-L4) • The femoral nerve is the largest nerve of this plexus, supplying muscles and skin on the anterior aspect of the thigh. It descends through the psoas major muscle and emerges low at its lateral border coursing inferiorly between the iliacus and psoas major muscle to enter thigh under the inguinal ligament. At the inguinal ligament (line running between anterior superior iliac spine and medial pubic tubercle) and just distal to it (in the femoral triangle) the nerve lies slightly deeper (0.5 to 1 cm) and lateral (approximately 1.5 cm) to the femoral artery, the vein is medial to the artery ("VAN"). At the femoral (inguinal) crease ( a few centimeters caudad to the inguinal ligament), the nerve lies underneath the fascia iliaca (iliopectineal fascia), deep to the fascia lata. Beyond the femoral triangle the nerve branches into anterior (quite proximally ) and posterior divisions. The anterior division gives muscular branches to the pectineus and Sartorius muscles and cutaneous branches (intermediate and medial cutaneous nerves of the thigh) to the skin on the anterior aspect of the thigh. The posterior division sends muscular branches to the quadriceps femoris muscle and gives rise to the saphenous nerve, its largest cutaneous branch. The saphenous nerve follows the femoral artery, lying lateral to within the adductor (Hunter's subsartorial) canal and then crosses it it anteriorly to lie medial to the artery. Distal to the canal, the saphenous nerve leaves the artery to lie superficial at the medial aspect of the knee. The nerve then continues inferiorly (subcutaneously) with the long (great) saphenous vein along the medial aspect of the leg down to the tibial aspect of the ankle. The saphenous branch supplies the skin on the medial aspect of the leg below the knee and on the medial aspect of the foot; it provides articular branches to the hip, knee and ankle joints. (Barash pg 953) Regarding A femoral artery and femoral nerve are located within the same fascial sheath - false Regarding D Sural nerve is a branch of femoral nerve - false • Sural nerve is made up of branches from the common peroneal (lateral sural) and tibial nerve (medial sural)

21. 21.What is true of lumbosacral plexus regional anatomy? a) Femoral artery and femoral nerve are located within the same fascial sheath. b) The obturator nerve is located between the adductor brevis and adductor magnus. c) Femoral artery is important in locating the saphenous nerve. d) Sural nerve is branch of the femoral nerve.

Consensus Answer C - 6 schools Answer B - 5 schools Answer D - 3 schools Answer C • There were no clear answers from our texts • This is based off of Cochrane collaboration reviews NNT: o Amitryiptyline -1.3 o Gabapentin - 5.8 o Duloxetine - 6.0 o Tramadol - 3.8 • Miller mentions the number needed to achieve at least 50% pain relief with antidepressants and gabapentin are 3.4 and 2.7 respectively (Miller chapter 32 page 916) • Barash lists all the above drugs as treatment for diabetic neuropathy with the first choice drug as serotonin NEPI reuptake inhibitors (duloxetine,milracipran). No mention of NNT (Barash pg 1647)

26. In Diabetic Neuropathy which of the following list would have the highest NNT? a) amitryptiline b) Gabapentin c) Duloxetine d) Tramadol

Consensus B Answer B - C3 1. Glottic level (Barash 1185) • Adult - C5-C6 • Full term infant - C4 • Premature infant - C3

27. Where is the location of the preterm neonates larynx? a) C2 b) C3 c) C4 d) C5

Answer B - Vasopressin o All the above drugs will cause efferent constriction with the exception of vasopressin will cause afferent constriction and thus decreases GFR o In the face of decreased afferent arteriolar pressure or blood flow, low levels of catecholamines, angiotensin, and arginine vasopressin (AVP) induce preferential efferent arteriolar constriction, which maintains glomerular filtration pressure. This is reflected by an increase in calculated filtration fraction (FF), which is the GFR expressed as a fraction of the renal plasma flow (RPF)—FF = GFR/RPF. o High levels of catecholamines and angiotensin (but not AVP) increase afferent arteriolar tone and decrease glomerular filtration pressure (and GFR) out of proportion to renal plasma flow, thus decreasing FF. (Miller chapter 23 pg 549)

28. All of the following cause decreased GFR and Afferent glomerular vasoconstriction EXCEPT: a) epinephrine b) vasopressin c) norepinephrine d) angiotensin

Answer C DKA is caused by high glucose levels exceeding the threshold for renal tubular reabsorption, creating osmotic diuresis and hypovolemia. Coupling between hepatic gluconeogenesis and ketogenesis leads to overproduction of ketoacids. An AGMA is created due to an increase in ketoacid formation. Hyponatremia results from the effect of hyperglycemia and hyperosmolarity on water distribution. Total-body potassium depletion is usually 3-5mEq/kg. Treatment is begun with an insulin bolus of 0.1units/kg and then an infusion of 0.1units/kg/hr. This is continued until normal acid-base status is obtained. Insulin infusion is reduced when blood glucose is controlled, pH >7.3, and HCO3 >18. Comparing and contrasting DKA and HHS are tables 19-4 and 19-5 in Coexisting. Hyperosmolarity >320mOsm/L is a feature of HHS but not of DKA. 300mg/dL is equal to 16.6mmol/L.

30. In regards to Diabetic Ketoacidosis, all are required for the diagnosis except: a) HCO3 <18 b) pH <7.3 c) Osmolality >320mosm/L d) Serum glucose > 16.7mmol/L

Answer C dilates coronary arteries - True • Verapamil has a major depressant effect on the AV node, a negative chronotropic effect on the SA node, a negative ionotropic effect on cardiac muscle and a moderate vasodilating effect on coronary and systemic arteries (P&P 5th ed pg 864) • Used in vasopastic angina pectoris and essential hypertension (mild vasodilating effects) Regarding A action at the level of the sodium channel - False • Primarily known as Calcium channel blocking drug • Note this answer is also somewhat true as there are two isoforms in which one acts on Na fast channels however C seems to be the better answer o The dextroisomer of verapamil is devoid of activity at slow calcium channels and instead acts on fast sodium channels, accounting for loacal anesthetic effects of verapamil (1.6 x as potent as procaine) o The leveoisomer of verapamil is specific for slow calcium channels, and the predominance of this action accounts for the classification of verapamil as calcium blocking drug. • (P&P pg 864) Regarding B Useful to control ventricular arrhythmias - False • Major depressant effect on AV node, a negative chronotropic effect on SA node • Verapamil treats SVT's Regarding D diminished conduction via accessory channel in WPW - False • Verapamil may precipitate ventricular dysrhythmias in patients with WPW (P&P pg 864) • Treatment of antidromic AVNRT is intended to block conduction of the cardiac impulse along the accessory pathway. Drugs that slow AV nodal conduction, such as adenosine, calcium channel blockers, B blockers, lidocaine and digoxin, may increase conduction along the accessory pathway and are contraindicated (CoE Chapter 4 pg 84) Regarding E decreases ventricular premature rates - False

3. Verapamil has which of the following cardiovascular effects a) Action at the level of the sodium channel b) Useful to control ventricular arrhythmias c) Dilates coronary arteries d) Diminished conduction via the accessory channel in WPW e) Decreases ventricular premature beats

Answer C When volume of distribution is small, clearance is responsible for the drop in plasma concentration. When VD is high, redistribution accounts for loss of analgesic effects. Barash p.508 As the drug with the smallest VD, remifentanil is the least dependent on redistribution. This is also appreciated in its very low context-sensitive half-time.

33. The termination of effect of which of the following opioids is least dependent on redistribution? a) Morphine b) Fentanyl c) Remifentanyl d) Meperidine

answer:C

34. A woman with 2 previous C-sections presenting with placenta previa has what risk of placenta accrete? a) 3% b) 10% c) 40% d) 70%

Answer C A PE (in this case likely a CO2 embolus) will cause hemodynamic collapse due to poor LV preload, RV dysfunction and TR secondary to increased PVR, desaturation due to poor pulmonary blood flow, but with a normal airway pressure. Regarding A and D, both of these will result in increased airway pressures. Regarding B, very nebulous term given the number of drugs that can be regarded as "anesthetics", but with respect to any of them, hypotension would be normal, airway pressures would remain normal, and PVR and TV should not be affected. Saturations should remain normal assuming controlled ventilation. In the case of a massive overdose causing very significant cardiac depression, poor SpO2 could be seen, but this would happen gradually, not suddenly as stated in the question.

39. After induction of general anesthesia for a lap chole, there is sudden hypotension, desaturation and the airway pressures remain normal. TEE shows enlarged RV with severe TR. What is the diagnosis? a) Anaphylaxis b) Anesthetic overdose c) Pulmonary embolism d) pneumothorax

Consensus A Answer A if you choose the axillary site, the left side is preferable over the right - True • Axillary artery - the tip of the 15-20 cm catheter may lie within the aortic arch; the use of the left axillary artery is recommended to minimize the risk for cerebral embolization during flushing (Kaplan chapter 14 pg 419) • The axillary artery has the advantages of patient comfort, mobility and access to the central arterial pressure waveform and complications are similar in incidence to those for use of radial and femoral arteries. • The risk of cerebral embolization is significantly increased when more centrally located vessels are used (therefore this suggests using left > right) (Miller chapter 45 pg 1350) Regarding B the placement of an ulnar artery catheter is contra-indicated in the case of a failure to insert the catheter in the ipsilateral radial artery - False • The ulnar artery is cannulated in a manner similar to that for the radial and has been used safely even following failed attempts to access the ipsilateral radial artery (Miller chapter 45 pg 1350) Regarding the brachial artery offers collateral circulation to the hand - False • The brachial artery, although lacking collateral branches to protect the hand, has a long track record of safe use • Bazaral and associated reported more than 3000 brachial artery catheters in patients undergoing cardiac surgery, with only one significant thrombotic complication and no long term sequalae (Miller chapter 45 pg 1350) Regarding - the femoral artery pressure curve doesn't look like the aortic pressure curve • Femoral artery is the largest vessel in common use for monitoring, but its safety appears to be comparable to those for other sites. • As with axillary artery pressure monitoring, the femoral artery waveform more closely resembles aortic pressure than do waveforms recorded from peripheral sites.

4. In a situation where the installation of a radial arterial catheter is impossible a) If you choose the axillary site, the left side if preferable over the right b) The placement of a ulnar artery catheter is contra-indicated in the case of a failure to insert the catheter in the ipsilateral radial artery c) The brachial artery offers collateral circulation to the hand d) The femoral artery pressure curve doesn't look like the aortic pressure curve

Answer: B CVP unchanged, CO increases 50%, EF increases; SVR decreases 20%, not 30%.

41. Pregnancy- which is the correct change? a) SVR decrease 30% b) CVP unchanged c) EF unchanged d) CO increase 30%

Answer: B

42. With regards to normal maternal physiology which one is correct? a) Increased Factor II and Factor V b) Decreased Factor XI and XIII c) increased prothrombin time and partial thrombin time d) Decreased Plasiminogen factor

Answer A Management requires 4 sequential steps: 1)lung isolation; 2)resuscitation; 3)diagnosis; 4)definitive treatment Regarding B, C, and D, massive hemoptysis is defined as expectoration of >200mL of blood in 24-48hrs. Commonest causes are carcinoma, bronchiectasis, and trauma (blunt, penetrating, or secondary to PA catheter). Death can occur quickly as a result of asphyxia. There is no consensus regarding what is the best method of lung isolation. Bronchial blockers usually pass easily into the right mainstem bronchus and are useful for right-sided hemorrhage (90% of PA-catheter induced hemorrhages are right-sided)

44. Pulmonary hemorrhage, which is false? a) Lung isolation might not be required if an interventional radiologist is immediately available b) Cause of death by asphyxia c) 90% of pulmonary hemorrhages caused by PAC are right-sided d) Massive hemoptysis is defined as 200mL over 24-48h

Answer B? So based on the available options, sulfur hexafluoride increases IOP the most. Barash p.1405 and 1415 discuss the use of sulfur hexafluoride for retinal detachment as a way of mechanically helping to reattach the retina due to increases in IOP, but they do not give a specific number with respect to how much IOP is increased. Most discussions are with respect to concomitant use of nitrous, but again, no specific numbers are given. Regarding A, rocuronium and IOP are not discussed, but from a BJA article, rocuronium does not have an effect on IOP. BJA 1999: 82:757-60. Regarding IOP, Miller p.2513-14 discusses this in a degree of detail. Hypercapnia causes choroidal congestion which can rapidly increase IOP. Coughing/straining/vomiting increases IOP to 30-40mmHg, and endotracheal intubation can cause similar increases. These increases are transient and relatively innocuous in a closed eye. In an open eye, this can cause loss of the globe's contents. Extrinsic compression of the eye increases IOP - a normal blink increases IOP by 10mmHg and a forceful lid squeeze increases it by 50mmHg. Deep inhaled or IV anesthesia can reduce IOP 30-40%. Opioids and atropine have little effect. Ketamine causes a modest increase. Sux increases IOP by 6-12mmHg and lasts 5-10 mins.

46. What increases intra-ocular pressure the most? a) Rocuronium at dose of ED95 b) Injection of hexafluoride c) ? d) ?

Answer D MAC and anesthetic effect is related to partial pressure, not volume % of anesthetic. Therefore for traditional vaporizers where flow-splitting occurs at the exit of the vaporizing chamber, these vaporizers are pressure-compensated. They are calibrated to maintain a specific partial pressure. Their volume % will increase as ambient pressure decreases, but their partial pressure will remain constant. The desflurane vaporizers are more accurately referred to as gas blenders, which maintain a constant concentration of vapour output (volume %) not a constant partial pressure. As such, the desflurane volume % will need to be changed linearly with changing atmospheric pressure.

47. At atmospheric pressure of 0.5 atm, what to dial Desflurane vaporizer at to achieve same effect as 1 MAC (6%) at 1 atm? a) 3% b) 6% c) 9% d) 12%

This is perhaps supposed to be an EXCEPT question. Barash p.809-810 cites malposition, male gender, and increased BMI as all being risk factors for ulnar neuropathy. Malposition with pressure on the ulnar nerve is the most classic, and therefore the one I would pick if this is not an except question. Prolonged surgery is not mentioned as a risk factor, though prolonged bed rest following surgery is stated to be a risk factor. Barash p.809-810.

48. Risk factor for ulnar neuropathy? a) Male gender b) Prolonged surgery c) malposition d) BMI

Answer A No mention of magnesium as a treatment for postop shivering. Regarding B, Barash p.1574 states that this is the most effective medication (in conjunction with rewarming) for postop shivering. Regarding C, clonidine increases the range of temperatures not triggering thermoregulatory defenses. As such, it can cause hypothermia, but is effective against shivering (as is dexmedetomidine). Barash p.393. Regarding D, alfentanil alters the thermoregulatory thresholds such that body temperature must fall by up to 4 degrees before shivering will be initiated. As such it is effective against postop shivering. Barash p.899

50. Which one is least effective for Postoperative shivering? a) Magnesium b) Meperidine c) Clonidine d) Alfentanil

Answer D From Miller 7th ed. P.2186 which states that PaCO2 increases more in ASA II and III patients than ASA I patients, and that these findings have been documented in patients with COPD and in children with cyanotic congenital heart disease. No mention as to mechanism is made. No mention of advanced age is made.

55. Increase in PaCO2 with laparoscopy in all except a) ASA 3 patients b) COPD c) Cyanotic heart disease d) Advanced age

Answer: B. Alkalosis, hypocarbia, hypothermia, decreased 2,3 DPG, COHb, Fetal Hb all cause left shift

7. Left shift Hb-dissociation curve (Higher saturation for the same PaO2). Which one is the cause? a) Changing fœtal Hb for adult Hb b) Decreased 2,3-DPG c) Increased temperature d) Decreased pH

ANSWER: B (consensus) WHY: Best answer; D is false, no mention of A or C in our texts. Botulinum toxin type A treatment delays and reduces the frequency of surgical procedures on the lower extremities (for CP patients). Succinylcholine may be used because it does not cause hyperkalemia in these children, whose muscles have never become denervated. Pain and spasm are regular features postoperatively. Epidural analgesia is particularly valuable when major orthopedic procedures are performed. Occasionally, two epidurals at different spinal sites may be required for multilevel surgery. Epidural and intrathecal forms of morphine have been used to control this pain. Intravenous morphine and midazolam also have been successfully used. Oral benzodiazepines may be required to reduce the incidence and severity of muscle spasms but should be used with caution if combined with opioid analgesia.

70. Regarding patients with cerebral palsy: a) Increased risk of local anesthetic toxicity so should use a lower dose b) Often get botulinum toxin injections for muscle spasticity c) Reduced need for opioids because decreased sensitivity to pain d) Cannot use succinylcholine because of exaggerated hyperkalemic response

Answer: B - Tight glucose control Miller Ch. 67. Pg. 2084 Barash Pg. 1091 Kaplan Ch. 28 Pg. 845 A - Epigastric aortic ultrasound use - FALSE: Avoiding aortic atheroma which can lead to macro- and micro- emboli can be achieved through the use of transesophageal or epiaortic echocardiography, and is a key neuroprotective strategy. B - Tight glucose control - TRUE: Although hyperglycaemia is associated with a worse outcome following various neurologic injury, it is both difficult and harmful to try and control blood glucose after cardiac surgery. Even a single episode of severe hypoglycaemia can have deleterious neurological consequences. An investigation comparing tight glucose control (between 4.4 to 5.6 mmol/L) to conventional management (maintaining blood glucose under 11.1mmol/L) found worse outcomes in the tight glucose control group - specifically an increased incidence of stroke. C - Maintain higher mean blood pressure - FALSE: Studies comparing patients left with a lower mean blood pressure (one study had a minimum MAP of 50mmHg) to those with a higher MAP (targeted between 80-100 mmHg) during CPB found a higher incidence of both cardiac and neurologic complications in the lower MAP group. D - Use pulsatile instead of non-pulsatile perfusion - FALSE: Not mentioned by Miller. Barash mentions that non-pulsatile CPB has been implicated in renal dysfunction, and the production of ischemic metabolic byproducts. No mention of neurological injury/protection afforded by a pulsatile flow. Pulsatile flow does equate to increased survival for high-risk patients, and may result in less need for inotropes. Kaplan discusses pulsatile vs non-pulsatile flow, and cites a study (by Murkin et. al) which found no significant benefit of pulsatile vs. non-pulsatile flow on neurologic and neuropsychologic outcome. However, they go on to discuss that the jury is still out on the benefits (if any) of pulsatile CPB. There is no true physiologic pulsatile pressure generated by systems in clinical use, and Kaplan suggests that newer technologies may actually prove to be beneficial. Kaplan then cites a pig study which showed newer forms of pulsatile CPB resulted in less jugular venous desaturation during rewarming after hypothermic CPB, but Kaplan emphasizes that the current clinically available pulsatile technology has little evidence to support its use. I chose option B because there is much evidence to suggest that tighter glucose control can be deleterious, whereas there isn't much evidence to suggest that pulsatile flow is beneficial. Therefore the most unhelpful thing (which would actually be harmful) would be attempting tight glucose control.

88. Regarding neuroprotection during CPB. Which is not helpful? a) Epigastric aortic ultrasound use b) Tight glucose control c) Maintain higher mean blood pressure d) Use pulsatile instead of non-pulsatile perfusion

Answer: C, D, F Slinger's Thoracics Ch. 28 Pg. 400, Miller Ch. 66 Pg. 1983, 1999 Miller quotes a study that identified 4 independent risk factors for acute lung injury following pulmonary resection, and if you remove having a pneumonectomy, the remaining three are answer options A, B, and E. Given that there are 6 answer options generated from an original question that should have only had 4 options, there are a few possibilities: there are 3 correct statements which were constant in all iterations of the question, and 3 incorrect statements (only one of which appeared in various iterations of this question), or there are more than 3 correct statements, and more than 1 incorrect statement, which were randomly shuffled so that 1 wrong and 3 right options were present in each iteration of the question. I don't know what ROH stands for, and options C and D I could not find any support for, so I'm putting my money only 3 correct options: More likely in a patient who was using alcohol prior to their surgery - TRUE: This is one of four independent risk factors identified for acute lung injury after pulmonary resection. The other three are: 1) a pneumonectomy, 2) excessive fluid administration intraoperatively, and 3) high intraoperative ventilatory pressure index (combined airway pressure and time). Large amounts of IV fluids given intraoperatively - TRUE: Unnecessary crystalloid or colloid administration will exacerbate the degree of edema and hypoxemia following pneumonectomy, and can contribute to Postpneumonectomy Pulmonary Edema. Also one of the 4 risk factors for acute lung injury after pulmonary resection. Most likely to occur in a patient with a short stump - FALSE?: I can't find any mention of issues with a short stump in Slinger, Miller or Barash. Post op respiratory failure is more likely to occur in patients with a right compared with a left sided pneumonectomy, and is thought to be due to the resultant higher PA pressures post operatively following a right pneumonectomy. If a patient's stump (regardless of how short it is) dehisces, there can be the development of a bronchopleural fistula. Occurs most commonly 24-28hrs post op - FALSE?: Also, no mention that I can find of timelines or most common time for post-operative respiratory failure to develop. Part of the definition of acute respiratory failure is that it occurs acutely, resulting in hypoxemia or hypercapnia, or involves >24hrs of post-operative mechanical ventilation, or a need for re-intubation. High peak inspiratory pressure - TRUE: See above, one of the 4 risk factors for acute lung injury after pulmonary resection. The actual risk factor is a high ventilatory pressure index, which is derived from a combination of airway pressure and time. ROH? - Don't even know what this means/stands for. My best guess is it's an abbreviation for Alcohol, as in R-OH...

97. Post-op respiratory failure after pneumonectomy. All increase the incidence except a) More likely in a patient who was using alcohol prior to their surgery b) Large amounts of intravenous fluid given intraoperatively. c) Most likely to occur in a patient with a short stump d) Occurs most commonly 24-28 hrs post-op e) High peak inspiratory pressure f) ROH ??

Answer: A - cocaine intoxication Chestnut Ch. 36, Pg. 825-835, Ch. 54, Pg. 1204-1206 Cocaine intoxication - TRUE: Cocaine intoxication symptoms include hypertension, labile blood pressure, and tachycardia. Cocaine causes vasospasm, thrombosis, and endothelia injury, which may damage end organs. Chest pain is a common complaint among young abusers, as are ECG changes (including QT prolongation, bradycardia, and tachy-arrhythmias - SVT, VT, VF). Neurologic complications that can result from cocaine abuse include subarachnoid hemorrhage, intracerebral hemorrhage, cerebral vasculitis, or TIAs. Cocaine can induce seizures, but they are typically self limiting and treated only supportively or with benzodiazepines. Cocaine ingestion can lead to gut ischemia, ulceration, and perforation. Delayed gastric emptying due to the anticholinergic effects of cocaine can increase the risk for aspiration. Cocaine is not hepatotoxic, but some cocaine users have abnormal liver enzyme levels. Cocaine abuse can cause renal failure due to rhabdomyolysis, renal infarction, and impaired immune function. Hyperthermia can be a fatal complication from cocaine intoxication, as it impairs cutaneous vasodilation/sweating. This patient has the hemodynamic manifestations in keeping with cocaine intoxication: both the hypertension and tachycardia, as well as the "settling" of these values after presentation. Cocaine intoxication can mimic pre-eclampsia or eclampsia, particularly if patients present with headache, blurred vision, or seizures, in addition to the hemodynamic changes. Case reports have been published where the only distinguishing factor was the rapid resolution of symptoms without delivery of the fetus. This patient's abdominal pain could be explained by placental abruption or preterm labour, both of which are common complications associated with cocaine abuse. This patient's extremes of hypertension and tachycardia, and subsequent settling point towards cocaine abuse. In addition, the gestational age makes pre-eclampsia less likely. Pre-eclampsia - FALSE: The severity of hypertension with preeclampsia can vary from mild to severe, and is accompanied by proteinuria, elevation of serum uric acid above 5.5mg/dL or 0.33mmol/L, and in severe cases, evidence of hepatic dysfunction, hemoconcentration, and thrombocytopenia are seen. By definition, preeclampsia presents after 20 weeks gestation, and most frequently is seen during the third trimester near term. Neurologic symptoms in pre-eclampsia can present as severe headache, hyperexcitability, hyperreflexia, and coma. Visual disturbances including scotomas, amaurosis, and blurred vision may be present. From a cardiovascular standpoint, an increased cardiac output is seen, with a hyperdynamic left ventricle, and mild to moderately increased systemic vascular resistance. Hepatic manifestations usually present as epigastric or right upper quadrant pain. This patient is at 29 weeks gestation, and would be manifesting signs of severe pre-eclampsia (as well as early onset pre-eclampsia), however the presence of proteinuria would be required to make the diagnosis. Tachycardia is not a feature of pre-eclampsia, nor is the "settling" of the sympathetic overactivity. Hypertension in pregnancy - FALSE: Hypertension in pregnancy (or gestational hypertension) presents as a mild, isolated hypertension, typically in the third trimester of pregnancy. Opioid/alcohol withdrawal - FALSE: Alcohol withdrawal would manifest as nausea, vomiting, tachycardia, hypertension, arrhythmias, tremor, hallucinations, agitation, and seizures, usually within 6-48hrs after cessation of alcohol intake. Delirium Tremens manifests as agitation, disorientation, hallucinations, and fever, combined with autonomic instability. While this lady has hypertension and tachycardia, her headache and abdominal pain are not explained by alcohol withdrawal. Opioid withdrawal would manifest as sympathetic hyperactivity resulting in flu like signs and symptoms, such as fatigue, weakness, restlessness, rhinorrhea, perspiration, fever and diarrhea. Depending on the opioid being abused, the onset and duration of the withdrawal symptoms can vary; morphine and heroin withdrawal symptoms begin within 6-18 hrs after cessation, peak by 3 days and last for 7 to 10 days. Methadone withdrawal occurs later, beginning within 24-48 hrs after cessation, peak within 3-21 days, and can last for as long as 6-7 weeks. None of the symptoms presented in this patient are suggestive of opioid withdrawal. Renal calculus - FALSE: Symptoms of renal calculi during pregnancy include flank and abdominal pain, urgency, dysuria, nausea and fever. Urolithiasis can mimic ectopic pregnancy, preterm labour, appendicitis, pyelonephritis, and benign hematuria of pregnancy. Urolithiasis most commonly presents during the second or third trimester. Pain could account for some hypertension and tachycardia, however, the values in the question stem seem too extreme to simply be pain related (though their "settling" characteristic may suggest this). There is no association with headaches.

103. A pregnant patient presents at 29 weeks gestation with hypertension 220/140 tachycardia of 140, headache and abdominal pain. Settles to 140/85 and pulse 100 while awaiting blood results. What is your most likely diagnosis? a) cocaine intoxication b) pre-eclampsia c) hypertension in pregnancy d) opiod/alcohol withdrawal e) renal calculus

Answer: A - Cribriform not involved Barash Pg. 1370-1371 Cribriform not involved - FALSE: In a LeFort III fracture, the fracture line parallels the base of the skull, separating the midfacial skeleton from the base of the cranium. The fracture line passes through the base of the nose and the ethmoid bone in its depth, and through the orbital plates. The cribriform plate of the ethmoid may or may not be fractured. A LeFort III fracture is a major contraindication to a nasotracheal intubation, because the cribriform plate of the ethmoid bone may be involved, and an endotracheal tube can enter the cranial cavity if inserted nasally in the presence of such a fracture. Midface displaced posteriorly - TRUE: The midface is mobilized and often distracted posteriorly in a LeFort III fracture. Airway often secured with a tracheo while the patient is awake - TRUE? I'm not sure what a "tracheo" is meant to represent - an awake tracheostomy? an endotracheal tube? The guidance offered by Barash is that a LeFort fracture can cause pain OR mechanical obstruction OR Trismus which may prevent mouth opening, and any patient who cannot open their mouth while awake, should give you pause for consideration of your options for securing the airway. Pain, and early trismus can by overcome by a general anesthetic with muscle relaxant; the mouth can be opened and the patient easily intubated. Direct trauma to the mandibular condyle or zygomatic arch may cause fractures which interfere with normal function of the TMJ. Compression fractures of the condyles may also impede mouth opening. Longstanding trismus may result in edema and swelling of the muscles, resulting in a mechanical interference with opening which is not overcome by a general anesthetic. An awake nasal fiberoptic intubation (or a sedated or an asleep fiberoptic intubation) can be performed in these situations. As to the frequency with which an awake intubation is performed in LeFort III fractures, I cannot comment (as neither Barash nor Miller do), but given that option A is clearly false, I will call this a true statement. Fracture line separates skull base from midface - TRUE: In a LeFort III fracture, the fracture line parallels the base of the skull, separating the midfacial skeleton from the base of the cranium.

104. Lefort 3. All true except a) cribiform not involved b) midface displaced posteriorly c) airway often secured with a tracheo while the patient is awake d) fracture line separates skull base from midface

Answer: A - Pulmonary infection Slinger's Thoracics Ch. 40 Pg. 588 Pulmonary Infection - TRUE: the immediate inflammatory response in the lung may lead to delayed immune suppression, which increases susceptibility to an infection later on. Hypercarbia is the principal cause of morbidity - FALSE: There is no explicit mention that I could find of a principal cause of morbidity. It is true, however, that hypercarbia is seen on presentation, accompanied by tachypnea, hypoxia, wheezing, and on occasion, hemoptysis. Complications mentioned include shunting, ARDS, and an increased susceptibility to pulmonary infection. Immediate radiographic changes are seen - FALSE: Radiographic findings (both CT and CXR) can lag behind the clinical picture by hours. Chest Ultrasound may aid in the rapid diagnosis (here Slinger talks about an extended FAST exam, but from our context, a POCUS would suffice). Increase in pulmonary compliance - FALSE: Lung contusions can lead to the development of ARDS, which is ultimately characterized by stiff, non-compliant lungs. Pulmonary contusion results in an inflammatory reaction, which leads to increased capillary permeability, and pulmonary edema. Pain also leads to splinting and progressive atelectasis. None of these things are in keeping with an increased pulmonary compliance, and if anything, you would expect a decrease in compliance with most of these complications. No change in capillary permeability - FALSE: The primary pathology results from a loss of vessel integrity at the alveolar capillary membrane, leading to intraparenchymal and alveolar hemorrhage, and edema. Surfactant production decreases, and shunt increases. Alveoli fill with blood and edema, resulting in functionally 'smaller' lungs.

105. What is the most likely complication after a pulmonary contusion? a) Pulmonary infection b) Hypercarbia is the principal cause of morbidity c) Immediate radiographic changes are seen d) Increase in pulmonary compliance e) No increase in capillary permeability

Answer: C - Normal tidal volume Miller Ch. 16, Pg. 382-383, Barash Pg. 1442, 1507, Coexisting Ch. 10 Pg. 259 Assuming the question is referring to an acute C5/6 fracture dislocation. [Can quickly determine the level of a spinal cord lesion by assessing whether the deltoid, biceps, brachialis and brachioradialis muscles are flaccid - if so, there is involvement of the C5 segment, implying at least partial diaphragmatic paralysis. C4 segment involvement will require artificial respiration.] Spinal shock occurs acutely, and results in complete cessation of spinal cord function below the level of the lesion (flaccid paralysis, loss of visceral and somatic sensation, a paralytic ileus, and loss of vasopressor reflexes). Spinal shock can persist for a few days, up to 3 months. Coexisting defines it as the hemodynamic changes associated with injury to the cervical cord, including loss of sympathetic nervous system activity with a decrease in SVR, and bradycardia due to the loss of T1-T4 cardio-accelerator fibres, which typically lasts 1-3 weeks. Spinal shock is not to be confused with neurogenic shock, which is defined by Barash as hypotension and bradycardia resulting from the loss of vasomotor tone and sympathetic innervation of the heart as a result of functional depression of the descending sympathetic pathways of the spinal cord. This is seen after high thoracic and cervical spine injuries, and improves within 3-5 days. So unfortunately, our textbooks disagree on how spinal shock is defined exactly. Regardless, Barash has a sentence that clearly answer this question (though in a paragraph later, he muddies the waters, potentially admitting to a decreased TV even in C5 lesions). Tachycardia - FALSE: An acute lower spinal cord injury would result in compensatory tachycardia from an intact autonomic nervous system. However higher spinal cord lesions often result in bradycardia in response to various stimuli, such as a valsalva, positional changes, or increased intrathoracic pressure. Bradycardia results from unopposed vagal stimulation, and loss of the cardioaccelerator fibers which reside between T1 and T4. These patients are unable to mount a tachycardic response to hemorrhagic shock. Sweating - FALSE: Autonomic dysreflexia/hyperreflexia can result from noxious stimulation below the level of the spinal cord lesion, and is seen in up to 85% of patients with a complete cord transection above T5 after the resolution of spinal shock and with return of spinal cord reflexes. Typical offenders/triggers include bowel or bladder distension. The result is a dramatic rise in blood pressure, bradycardia (baroreceptor reflex), dysrhythmias, a marked reduction in flow to the periphery (vasoconstriction below the level of the lesion), and flushing and sweating (vasodilation) in the area above the lesion. Outside of an autonomic dysreflexia "spell" these patients have impaired thermoregulation - they can be vasodilated and unable to shiver, resulting in hypothermia. Similarly, they have an impaired sweating mechanism, and may develop hyperthermia. Under a general anesthetic, it is important to monitor temperature closely, as both hypo- and hyperthermia are concerns. Normal tidal volume - TRUE: Lesions between C5 and T7 cause significant alterations in respiratory function, owing to the loss of abdominal and intercostal support. Paradoxical respirations result from indrawing of flaccid thoracic muscles during inspiration. This reduces vital capacity by 60%, and a weak cough inhibits effective clearance of secretions, increasing the risk of infection and atelectasis. However, injuries at C5 or lower (according to Barash) are usually associated with normal tidal volumes because the function of the diaphragm is intact, whereas patients with injuries at C4 or above may require permanent ventilatory assistance. Barash does discuss paradoxical respirations leading to a reduction in tidal volume, though a precise level at which this occurs is not defined (just states that it happens in quadriplegics). Cold extremities - FALSE: The periphery is atonic, with a dilated peripheral vascular bed, which would result in warm extremities, not cold ones initially. Poikilothermia is likely to occur (which is an inability to maintain a normal/stable temperature), see explanation of sweating answer above.

106. Which of the following are consistent with spinal shock following a C5/6 fracture dislocation? a) tachycardia b) sweating c) normal tidal volume d) cold extremities

Answer: D - Resting ventilation Barash Pg. 924-926 Myocardial oxygen demand - TRUE: A decrease in preload results in a decreased cardiac output during a high spinal. Arterial dilation results in significant decreases in peripheral vascular resistance. A decrease in afterload and cardiac output is seen with a high spinal. Heart rate does not change significantly during spinal anesthesia in most patients, however clinically significant bradycardia occurs in 10-15% of patients, and increases in incidence as the level of the block rises. As a result of these changes, you would expect decreased myocardial oxygen demand. Coronary blood flow - TRUE: Arterial and venous dilation occurs with a high (or any) spinal. Venodilation increases volume in capacitance vessels, and decreases venous return and right sided filling pressures. Reduction in afterload is also seen due to peripheral vasodilation. These factors would contribute to a drop in coronary blood flow. Hepatic blood flow - TRUE: The abdominal organs derive their sympathetic innervation from T6 to L2. Blockade of these fibers results in unopposed parasympathetic activity by way of the vagus. Secretions increase, sphincters relax, and the bowel constricts. Parasympathetic tone would also vasodilate, resulting in decreased blood flow to abdominal organs due to ensuing hypotension. Resting ventilation - FALSE: A high block can result in abdominal and intercostal muscle paralysis, and can impair ventilatory functions that require active exhalation (ie NOT resting ventilation where expiration is passive). Expiratory Reserve Volume, peak expiratory flow, and maximum minute ventilation may be significantly reduced by high spinal and epidural blocks. In keeping with the previous question, the diaphragm should still be mostly intact with a T5 block, and ventilation should be minimally impacted. Despite this, patients with a high spinal or epidural block may complain of dyspnea, despite normal or elevated minute ventilation. This is likely due to an inability to feel the chest wall move during breathing.

107. T5 spinal is assoc with decreases in all except: a) Myocardial oxygen demand b) Coronary blood flow c) Hepatic blood flow d) Resting ventilation

Answer: B - 7th and 8th Rib fractures ATLS Ch. 4 Pg. 86-98 1st and 2nd Rib fractures - TRUE: Multiple rib fractures, or fractures to the first and second ribs indicate a high energy/severe force delivered to the chest and underlying tissues, and suggests there is a high likelihood of major vascular injury in the chest. The first two ribs are protected by the scapula, humerus, clavicle and upper limb muscles, so injury that results in the fracture of these ribs indicates a high energy force, putting the head, neck, spinal cord, lungs and great vessels at risk for serious associated injury. 7th and 8th rib fractures - FALSE: Ribs 4-9 are more likely to be fractured with blunt trauma. So fractures of ribs 7 and 8 are quite common to see. While a flail chest is by definition the involvement of 2 or more adjacent ribs (so this technically would count as a flail chest), there is a separate option for flail chest, so I would count this as a non-flail chest option. Widened mediastinum - TRUE: A widened mediastinum indicates there is a high likelihood of a major vascular injury in the chest such as an aortic disruption. This is one of the life threatening injuries that must be identified in the secondary survey (often because these signs are not seen until the secondary survey is performed). Large hemothorax - TRUE: A massive hemothorax is defined as more than 1500mL of blood, or one third or more of the patient's blood volume in the chest cavity. It is one of the life threatening injuries that must be identified in the primary survey of a person with a suspected/known thoracic injury. Even if considering a non-massive hemothorax, it is also described as a lethal injury that should be identified in the secondary survey. Flail Chest - TRUE: Flail chest occurs as a result of multiple (adjacent) rib fractures, which are an indication of a high energy force involved in the trauma. It is one of the life threatening injuries that must be identified in the primary survey.

108. Which is least indicative of a serious chest injury? a) 1st and 2nd rib fractures b) 7th and 8th rib fractures c) Widened mediastinum d) Large hemothorax e) Flail chest

Answer: B - Pupils react to light

109. All of the following are components of GCS, EXCEPT: a) Eyes open to pain b) Pupils react to light c) Following commands d) Spontaneous extension of limbs

Answer: A - Male gender Miller Ch. 89 Pg. 2625 OLD Miller Ch. 78 Pg. 2448 This is a repeat question from 2013 (which I've previously answered). Unfortunately, the major source of the answer is still an outdated textbook. Male gender - FALSE: Female gender is allegedly associated with delayed discharge, based on the table from OLD Miller. Spinal anesthesia - TRUE: prolonged motor blockade or impairment of joint position sense can lead to a delay in discharge. Opioid use intra-operatively - ?TRUE: The chart from OLD Miller does not mention use of opioids, though the most recent version of Miller states that premedication of children with Fentanyl is associated with delayed discharge. Could not find a source to back up this option, however the male gender option is clearly incorrect. Can mechanistically come up with many ways that increasing opioid use intra-operatively can translate into delayed discharge (PONV, sedation, respiratory rate depression, etc...). Volatile anesthesia - TRUE: Again, no clear statement that volatile anesthetics are the culprits, however "general anesthesia" which could (and frequently does) involve volatiles, is listed in the Old Miller table as a factor alleged to delay discharge. The section on delayed discharge is no longer present in the newest edition of Miller. The only places where delayed discharge is discussed in the new edition are during the discussion of paediatric anxiolytic premedication, where he mentions that benzodiazepines are no longer felt to delay discharge, but that premedication with Fentanyl does appear to contribute to delayed discharge. The other section where it is mentioned is the discussion of spinal anesthesia, and the desire to avoid delaying discharge from an inappropriate drug choice or excessive drug dose.

111. All are predictors of delayed discharge EXCEPT a) Male gender b) Spinal anesthesia c) Opioid use inter-op d) Volatile anesthesia

Answer: D - Gradual increase in the baseline CO2 Miller Ch. 29 Pg. 782 OLD Miller Ch. 44 Pg. 1427 Figure 44-16 Hall Comprehensive review, Ch. 1 PDF Pg. 45 (the old Hall, not the textbook-appearing new version we now have). Slow drop of the slope of the inspiratory phase - FALSE: This pattern is typically seen with an inspiratory valve that is stuck open during mechanical ventilation (Capnograph J). Oscillation of the plateau of the expiratory phase - FALSE: These are cardiogenic oscillations, which are often seen with sidestream capnographs in spontaneously breathing patients at the end of each exhalation. The heart beat causes back and forth motion between the exhaled and fresh gas, resulting in oscillations in the sampling line. (capnograph M) Slow rise of the expiratory phase slope - FALSE: This may be seen with spontaneous breathing, or with a prolonged exhalation during spontaneous breathing. I couldn't find a particular condition where there is a slow rise in the expiratory phase (which would be Phase II), but looking at the various capnograph examples given in OLD Miller, a slow rise in the expiratory phase is seen with spontaneously breathing patients (when comparing the expiratory upslope to mechanically ventilated patients), and in those spontaneously breathing and showing a prolonged exhalation. It's hard to differentiate between a slow rise in Phase II VS Phase III, as the inflection point is not always clear. According to Hall, a slanted expiratory upstroke (Phase II) can occur if there is partial airway obstruction (due to a kinked ETT/circuit, or due to COPD or acute bronchospasm), if a sidestream analyzer is sampling gas too slowly, or if the response time of the capnograph is too slow for the patient's respiratory rate. Gradual increase in the baseline CO2 - TRUE: The expiratory valve is most vulnerable to incompetence, as it has the most exposure to moisture. Incompetence of unidirectional valves will result in rebreathing of carbon dioxide. With an incompetent expiratory valve and mechanical ventilation, an increase in the baseline CO2 is seen (capnograph L). With spontaneous ventilation and an incompetent expiratory valve, the tracing in capnograph K is typical.

112. What will you see on the capnograph if you have an incompetent circuit expiratory valve? a) slow drop of the slope of the inspiratory phase b) oscillation of the plateau of the expiratory phase c) slow rise of the expiratory phase slope d) gradual increase in the baseline CO2

Answer: A - Neostigmine Coexisting Disease Ch. 9 Pg. 186, 188 Barash Pg. 1564 Neostigmine - FALSE: Theoretically, neostigmine administration (or administration of any anticholinesterase drugs) can precipitate bronchospasm secondary to stimulation of postganglionic cholinergic receptors in airway smooth muscle. This is not usually seen, as neostigmine is typically given to antagonize neuromuscular blockade, and is administered with an anticholinesterase, which likely offsets the effect through protective bronchodilation. Epinephrine - TRUE: Technically is a beta-2 agonist (amongst other receptors), so functions through that mechanism. Can be given as an inhaled racemic mixture to relax airway smooth muscles. If bronchospasm is life threatening, an IV epinephrine infusion yields profound bronchodilation. Beta-2 Agonists - TRUE: Bronchospasm due to asthma may respond to deepening of the anesthetic with volatile agents. If this does not work, administration of Beta2 agonists should be considered. In addition, treatment of bronchospasm in status asthmaticus is begun with continuous administration of Beta-2 agonists by inhalation Magnesium - TRUE: Can be used in intractable cases of status asthmaticus.

113. Regarding treatment of bronchospasm, which is false a) Neostigmine b) epinephrine c) B2-agonist d) Magnesium

Answer: D - Take her to the OR, resuscitate as necessary Barash Pg. 1673-1674 Miller Ch. 10 Pg. 236-237, 248 Negotiate with the patient as to which resuscitative measures are acceptable - TRUE: The only modalities that are not routine anesthetic care are cardiac massage and defibrillation. Therefore, the specific interventions included in a DNR status must be clarified with specific allowance made for methods necessary to perform anesthesia and surgery. Ask to temporary remove DNR status - TRUE: A mutual decision can often be reached to suspend or severely limit a DNR order in the perioperative period if the patient understands the special circumstances of perioperative arrest, that interventions are brief and usually successful, and that the physicians support the patient's goals in coming to surgery and values in desiring not to prolong death. Refuse to care for her and ask a colleague who's willing to do so - TRUE: Despite full information, patients may sometimes request or demand care that is unreasonable, either because it will adversely affect the performance of a surgical procedure, or because it would be associated with unreasonably high risks. When a patient demands a technique that is inappropriate or outside the realm of reasonable practice, the anesthesiologist is under no ethical obligation to provide that care. No physician can be compelled by a patient to practice negligently. Anesthesia care of patients can involve ethical controversy, legitimate disagreement, and moral ambiguity. The ASA, the British Medical Association, and the Hastings Centre all have issued statements recognizing the physician's right to withdraw from situations in which ethical standards of patient care are in serious conflict with his or her personal values. The ASA specifically recognizes conscientious withdrawal from patient care in the case of patients with DNAR orders or other directives that limit treatment. It is suggested that the entire health care team have a discussion prior to a "critical event" occurring, which would allow any conscientious objectors to withdraw from the health care team (and presumably be replaced by someone agreeing to participate under the stipulations acceptable to the patient). Take her to the OR, resuscitate as necessary - FALSE: Obviously inappropriate. Informed consent is meaningless if the patient cannot also refuse medical therapy because the consent process is then merely an exercise in the patient's acquiescence to the physician's will.

114. A patient with a small bowel obstruction requests no CPR during the operation. You don't feel comfortable going ahead, which of the following is not acceptable? a) Negotiate with the patient as to which resuscitative measures are acceptable b) Ask to temporary remove DNR status c) Refuse to care for her and ask a colleague who's willing to do so d) Take her to OR, resuscitate as necessary

ANSWER: D (Consensus D) Reference: Barash Ch 7 Pg 164 Background: Recall, a first-order kinetic process is one in which a constant fraction of the drug is removed during a finite period of time regardless of the drug amount or concentration. Regarding A: see table, true Regarding B: as per definition above, a constant fraction is removed per unit of time in first-order kinetics Regarding C: the absolute amount of drug removed is proportional to the concentration of the drug. When the concentration is high, more drug will be removed than when it is low. Regarding D: more of an incorrect technicality. The key is the statement that a constant fraction of the drug is removed during a finite period of time regardless of the drug amount or concentration. Although, note that the rate of change of amount of drug at any given time is proportional to the concentration present at that time. However, I guess when this question stem said "speed" it means the rate constant (which is always constant).

115. About 1st order kinetics, which is false a) after 5 half-lives, 97% is eliminated b) constant fraction is eliminated c) If drug quantity increases, the quantity eliminated in absolute increases d) If drug quantity increases, the speed of elimination decreases.

ANSWER: C (Consensus mainly C, some B) Reference: Barash Ch 33, Pg 894 Regarding A, B, and D: (Barash Pg 894) "During the drug redistribution phase the blood concentration typically is higher in older patients, partly because of a mildly contracted blood volume and partly because the reduction in muscle mass slows removal of the drug from blood. " This description would also suggest there is less redistribution that occurs, hence making B a wrong answer. Regarding C: Barash Pg 895 states that total blood flow to any organ does NOT appear to decrease beyond that expected from the decrease in organ mass.

116. Which does not explain a high peak drug level in a elderly patient? a) decreased muscle mass b) decreased redistribution c) decreased perfusion to highly perfused organs d) decreased blood volume

ANSWER: D (Consensus D) assuming B says PHTN Reference: Barash Ch 22, Pg 595 Regarding A: OSA is a syndrome defined by periodic obstruction of the upper airway during sleep, leading to episodic oxygen desaturation and hypercarbia. This episodic desaturation, in turn, causes episodic arousal, leading to chronic sleep deprivation with daytime hypersomnolence and even behaviour changes in children. Regarding B: Depending on the frequency and severity of events, it may lead to other changes such as chronic pulmonary hypertension and right heart failure. Regarding C: As per Barash, one should discuss whether it is feasible to perform surgery under neuraxial, regional, or local anesthesia, decreasing the total amount of anesthesia or opioids needed. Miller (Pg 1111 Ch 38) also states that patients with OSA are more sensitive to the respiratory depressant effects of opioids. Not specific to OSA, but in Coexisting Disease (Ch 16, Pg 326) it states: "According to the ASA Practice Guidelines, local or regional anesthesia should be the primary anesthetic choice for obese patients undergoing surgery, with general anesthesia used only when necessary." Regarding D: When procedures are performed on an outpatient basis, prolonged postoperative monitoring should be continued to ensure that the patient is able to maintain room air saturation without obstruction when left undisturbed in recovery. This suggests that OSA is not a contraindication to outpatient surgery, but may need more monitoring.

117. Concerning OSA, which is false? a) Daytime hypersomnolence b) Associated to HTN c) Neuraxial anesthesia is preferable d) Is a contraindication to ambulatory surgery

ANSWER: A (Consensus A, some C) (Miller Ch 100, Pg 3016) Ischemic Optic Neuropathy (ION) has two types - anterior (AION) and posterior (PION). They can be arteritic or nonarteritic. Spontaneously occurring ION, unrelated to surgical procedures, is usually caused by AION. The overwhelmingly type found perioperatively is nonarteritic ION (more common than arteritic). It has been reported after a wide variety of surgical procedures, with most after cardiothoracic surgery, instrumented spinal fusion operations, head and neck surgery, orthopedic joint proceudres, and surgery on the nose or sinuses. Regarding A: Specifically regarding lumbar spine surgery, from a retrospectively controlled study, 6 specific factors were found to confer higher risk for perioperative ION: male gender, obesity, positioning on a Wilson frame, duration of anesthesia, large blood loss, and a relatively low ratio of colloid to crystalloid fluid resuscitation. This is very confusing, but it must mean that less colloid is a risk factor, therefore using colloid is NOT a risk and appears protective based on the table of ORs. Regarding B, C, D: (Miller Ch 100, Pg 3022) Factors in perioperative ION via retrospective case-control studies: long surgery, hypotension, blood loss, anemia or hemodilution, altered venous hemodynamics, flow of cerebrospinal fluid in the optic nerve (including influence of patient positioning and perioperative fluid resuscitation), abnormal autoregulation in the optic nerve, anatomic variants in blood supply to the optic nerve, male gender, small cup-to-disc ratio, use of vasopressors, presence of systemic vascular risk factors (including HTN, DM, atherosclerosis, hyperlipidemia, obesity, and smoking history), prone positioning, lengthy surgery for spinal fusion, nature of the intravascular fluid resuscitation during surgery, and other preexisting systemic abnormalities (sleep apnea syndrome and hypercoagulability).

118. Not a risk factor for ION ? a) Colloïde b) Ventral position c) Hypotension d) Bleeding

ANSWER: A (Consensus A) Reference: Barash Ch 24 Pg 675 The major degradation product of Sevoflurane interaction with CO2 absorbents is compound A (fluoromethyl-2, 2-difluoro-1-(trifluromethyl) vinyl ether). Factors apparently leading to an increase in compound A: 1) Low flow or closed circuit anesthetic techniques (opposite of Option A) 2) Baralyme (no longer available) rather than soda lime 3) Higher concentrations of sevoflurane in the anesthetic circuit (Option D) 4) Higher absorbent temperatures (Option B) 5) Fresh absorbent (Option C)

119. Production of Compound A from sevoflurane will be increased by all except? a) High fresh gas flow. b) Increased temperature. c) Change of CO2 absorbent for a new one. d) High concentrations of sevoflurane.

Answer: B Cote: 132 B: Fentanyl's Vdss is approximately 5.9 L/kg in term neonates, and decreases with age to 4.5 L/kg during infancy, 3.1 L/kg during childhood, and 1.6 L/kg in adults.1300 A: This increased Vdss results in a lower blood concentration after bolus administration in neonates and infants. Administration of fentanyl 3 μg/kg by slow IV push in term infants (1 to 7 months age) intra-operatively neither depressed respiration nor caused hypoxemia in a placebo- controlled trial. C: Clearance matures with gestational age; 7 mL/min/kg at 25 weeks PMA, 10 mL/min/kg at 30 weeks PMA, and 12 mL/min/kg at 35 weeks PMA.1296 The clearance of fentanyl is reduced to 70% to 80% of adult values in term neonates and, when standardized to a 70 kg person, reaches adult values (approx. 50 L/hr/70 kg) within the first 2 weeks of life.858 Clearance of fentanyl in older infants (more than 3 months of age) and children is greater than that in adults when expressed as per kilogram (30.6 mL/kg/min vs 17.9 mL/kg/min, respectively), resulting in a reduced elimination half-life (68 minutes vs 121 minutes, respectively) D: fentanyl is metabolized by the liver, and small amount is filtered through the kidneys unchanged. Renal function is depressed in children compared to adults.

12. The following pharmacokinetics of an equivalent dose of fentanyl in infants compared to adults is TRUE: a) Higher peak plasma b) Higher total volume of distribution c) Shorter T1/2 d) Higher renal excretion

ANSWER: C (Consensus C, some A) Reference: Barash Ch 23 Pg 634, Miller Ch 38 Pg 1131, Coexisting Disease Ch 21 Pg 456 Regarding A: (Barash) The incidence of atlantoaxial subluxation may exceed 40% in RA patients and flexion of the neck can compress the spinal cord. Technically it's flexion not extension. Regarding B: Nothing mentioned in Barash/Miller/Coexisting regarding the prevalence but seems like this statement should be true...and C is definitely false. Regarding C: (Barash) The degree of cord compression may not correlate with the patient's symptoms. (Miller) The disease duration, severity, and symptoms do not correlate with cervical spine subluxation. Regarding D: (Miller) The specific radiographs required are anteroposterior and lateral cervical spine films with flexion, extension, and open-mouth odontoid views. Significant abnormalities (ie anterior atlas-dens interval > 9 mm or posterior interval < 14 mm) require consultation with a neurologist or neurosurgeon. (Coexisting Pg 456) Radiographic demonstration that the distance from the anterior arch of the atlas to the odontoid process exceeds 3 mm confirms the presence of atlantoaxial subluxation.

120. Regarding patients with RA, which one is false? a) Neck extension will increase atlanto-axial distance. b) Severe consequence such as myelopathies, quadriplegia and death are rare. c) Absence of symptoms correlates with lack of instability. d) A 3 mm difference on (posterior?) alignment is significant.

ANSWER: D (Consensus D, one A, one B) Regarding A: (Barash Ch 3 Pg 76) HBV may be infectious for at least 1 week in dried blood on environmental surfaces. (Miller Ch 110 Pg 3236) HBV remains viable on needles, environmental surfaces, and gloves for longer than 14 days. Regarding B: (Miller Ch 61 Pg 1856, Table 61-10) Also direct statement: "Ninety percent of posttransfusion hepatitis is caused by the hepatitis C virus." Note that this is despite the lower % risk of transfusion-transmitted infection for Hep C vs Hep B. Regarding C: (Barash Ch 3 Pg 76) After HCV seroconversion, only 15-25% will clear the virus spontaneously. Of those that develop chronic hepatitis, 20% will develop cirrhosis over the following 20 to 30 years and 1-2% of those will be diagnosed with hepatocellular carcinoma. (Miller Ch 61 Pg 1857) The CDC reports that of every 100 people infected with HCV, of course 60 to 70 will go on to develop chronic liver disease. Regarding D: (Miller Ch 110 Pg 3236) HBV is hardy. Neither treatment at 60°C for 4h nor the use of disinfectants containing phenol or chlorine is sufficient to inactivate the HBV particle with certainty.

121. Regarding viral hepatitis, which one is false? a) HBV can persist and remain infectious in the environment for up to 14 days. b) HCV is the principal agent of transfusion-acquired hepatitis. c) Most people infected with HCV will develop a chronic hepatitis. d) HBV is susceptible to chlorine-containing disinfection solutions.

ANSWER: B (Consensus B) References: Barash Ch 38 Pg 1087, Miller Ch 67 Pg 2053 Definitely goals of MS: maintain preload (A), decrease PVR (C), decrease HR (D). Regarding B, Barash just says to prevent increase it and that vasopressors may be required for systemic hypotension. Miller discusses that that it may be appropriate to lower SVR for a LV that has marked systolic dysfunction, but care must be exercised because a reduction in afterload is inevitably accompanied by a reduction in preload, which may not be desirable in patients with significant MS.

122. All of the following are hemodynamic goals for severe mitral stenosis EXCEPT: a) maintain preload b) afterload reduction c) decrease pulmonary vascular resistance d) slower heart rates

ANSWER: A (Consensus A) References: Barash Ch 55 Pg 1595), Miller Ch 101 Pg 3044 Regarding A and D: A large, randomize, prospective trial found that a strategy that used tidal volumes of 6 mL/kg or less and maintained static (plateau) airway pressure at ≤ 30 cm H2O resulted in a relative mortality reduction of 22% when compared to a control group ventilated with tidal volumes of 12 mL/kg. This approach is the only intervention that has been unequivocally proven to reduce mortality in patients with ARDS. Regarding B: Inhaled nitric oxide variable and transiently improves oxygenation in ARDS by improving blood flow to ventilated alveoli. Several randomized, prospective trials have failed to show any relevant long-term outcome benefits. It may still be used as "rescue" therapy in selected patients with severe, refractory hypoxemia. Regarding C: (Miller Ch 101 Pg 3044) Regarding tracheotomy in general (not specific to ARDS), the timing of tracheotomy based on a meta-analysis did not alter mortality or increase the risk for hospital-acquired pneumonia.

123. In ARDS, what decreases mortality? a) Tidal volumes of 6ml/kg b) Inhaled NO c) Early tracheostomy d) Tidal volumes of 12 ml/kg

ANSWER: C (Consensus mainly C) Reference: CAS Guidelines Pg 95 Background: Monitoring guidelines for standard patient care apply to all patients receiving general anesthesia, regional anesthesia, or intravenous sedation. Required monitors must be in continuous use throughout the administration of all anesthetics. The following are required: ♣ Pulse oximeter ♣ BP (directly or noninvasively) ♣ ECG ♣ Capnography for general anesthesia and sedation (RSS 4-6) ♣ Agent-specific anesthetic gas monitor, when inhalational anesthetic agents are used Hopefully this is just a question based on the required monitors from the CAS guidelines. Otherwise, it is not so straightforward. Note that according to discussion in Barash Ch 32 and Miller Ch 90, the ASA does not currently require capnography for NORA (but of course it is very desirable). Regarding B (? Also true?) : Miller Ch 90, Pg 2652 states that "No matter what type or level of anesthetic care is provided, an anesthesia machine should be present (or readily available) in all NORA locations because conversion to general anesthesia is always a possibility. If the room cannot accommodate anesthesia equipment, it is not an acceptable site for the delivery of anesthesia. Regarding A: not discussed in Miller/Barash/CAS guidelines Regarding D: Not required, as per table above, you just need easy and expeditious access to the patient

124. Out Of OR, deep sedation, which is required a) Presence of anesthesia tech b) Anesthesia machine and piping c) Capnography d) Unobstructed access to patient

ANSWER: D (Consensus D) Reference: Barash Ch 16 Pg 441 Regarding A: Desmopressin (DDAVP) is a synthetic analog for the endogenous antidiuretic hormone, vasopressin. It acts at the V2 receptor, which is found in the nephron and within endothelial cells. Mechanistically it causes the release of FVIII and vWF from within vascular endothelial cells thereby improving platelet function. DDAVP does not increase the synthesis of these factors - tachyphylaxis occurs with repeat dosing. Regarding B: The hemostatic potential for DDAVP has been vastly studied for patients with critical bleeding from cardiac or spine surgery, uremia, antiplatelet agents, and liver disease. However studies have not shown significant outcome improvements. The only consistent clinical benefit of DDAVP is improved bleeding times for patients with congenital or acquired platelet dysfunction from cardiopulmonary bypass, chronic renal failure, or aspirin therapy. Regarding C: The appropriate dose for hemostasis is 0.3 mcg/kg IV over 20-30 min. The effects last for ~6-8 h. Regarding D: Hyponatremia and water retention are rare complications but have been reported in pediatric patients. Hypotension is the most commonly reported side effect (presumably secondary to arterial vasodilation from the release of NO)

125. DDAVP a) Acts on V1 and releases VWF from endothelium b) Used to treat bleeding in liver failure and ureamic patients c) Dose is 30 mcg/kg/min d) May lead to hyponatremia

Regarding A: included in Box 55-5 BUT there is no discussion in the 2015 ACLS guidelines regarding hand position of compressions in pregnant patients. Regarding B: no. Also from 2010 (last update for this item), internal or external fetal monitors can be removed during resuscitation. Regarding C: (ACLS) aortocaval compression can occur for singleton pregnancies at approximately 20 weeks of gestational age, at about the time when the fundus is at or above the umbilicus. Although chest compressions in the left lateral tilt position are feasible in a manikin study, they resulted in decreased CPR quality (less forceful chest compressions) than is possible in the supine position. Manual left lateral uterine displacement (LUD) effectively relieves aortocaval pressure in patients with hypotension. Note: the 2015 guidelines removed the lateral tilt technique, and only recommends manual LUD.

128. CPR in pregnant a) Compressions slightly higher than mid sternum b) Fetal heart monitoring c) Lateral tilt after 28 weeks d) ??

ANSWER: C (Consensus mostly C is the most correct answer, but some said B IF this was an EXCEPT question) References: Miller Ch 81 Pg 2451-2, ATLS Ch 8 Pg 216, Barash Ch 52 Pg 1514 Background: (Miller) Acute compartment syndrome of the extremities is defined as a "condition in which increased pressure within a limited space compromises the circulation and function of the tissues within that space. Regarding A: no discussion in Miller/Barash/ATLS manual regarding outcomes of fasciotomy. Miller states that prophylactic fasciotomy may decrease subsequent muscle loss. Barash states that emergency fasciotomy can be effective in preventing irreversible muscle and nerve damage. Nothing to indicate definitely good outcome. Regarding B: (ATLS) Tissue pressures that are greater than 30-45 mmHg suggest decreased capillary flow, which may result in increased muscle and nerve damage caused by anoxia. Another method is the "delta-P" method, where the compartment pressure is subtracted from the diastolic blood pressure, yielding the delta-P. If this value is 30 mmHg or less, this suggests that the patient may have a compartment syndrome. Regarding C: (Miller) "The most common cause of compartment syndrome is edema secondary to muscle injury and associated hematoma formation." Regarding D: (Miller) The most common fractures associated with the development of compartment syndrome are those of the tibial shaft (40%) and forearm (18%). Forearm compartment syndrome requiring fasciotomy predominantly affects males and can occur after either a fracture or soft-tissue injury. A further 23% of cases are caused by soft tissue injuries without fracture. Guess it depends on your definition of "commonly".

129. Compartment syndrome a) Fasciotomy has good outcome b) Indication of compartment syndrome with pressure monitoring > 20cmH20 c) Commonly secondary to edema and muscle swelling and hematoma d) Common occurs in the absence of a fracture

Consensus A Answer A Preclampsia • HELLP may be a variant of severe preeclampsia, but this is controversial because a substantial fraction of HELLP syndrome patients do not have hypertension or proteinuria. • HELLP syndrome is characterized by hemolysis, elevated liver enzymes, and a low platelet count. • Chestnut pg 840 Regarding B hemolytic syndrome and D Fatty liver • HEELP can mimic these syndromes o Diagnosis can be especially challenging because numerous medical and surgical disorders, including acute fatty liver of pregnancy, hemolytic uremic syndrome, thrombotic thrombocytopenic purpura, idiopathic thrombocytopenic purpura, and lups, can mimic HELLP syndrome. Regarding C PE • Nothing specifically mentioned • Following shows table with patients with HELLP and associated complications

13. Wich one is associated with HELLP? a) Preeclampsia b) Hemolytic uremic syndrome c) PE d) Fatty liver of pregnancy

ANSWER: Probably should be EXCEPT A (D and C are definitely true, B is kind of) (Consensus Answers divided mainly upon correct answer = C, or EXCEPT A) Reference: Cote Ch 31 Pg 661-3 Regarding A: (when is codeine ever the recommended analgesic?) Codeine is metabolized by cytochrome P450 CYP2D6 to its active analgesic metabolites. The CYP2D6 gene displays polymorphism, including gene duplication (ultra-rapid metabolizers) and inactive genes. Ultra-rapid metabolizing genotype is present in 3% of Caucasians, 10-30% of Arabian and Northeast African populations. Almost 10% of children lack CYP2D6. This variability in codeine metabolism makes it less than ideal for children with OSA. Regarding B: Allowing spontaneous respirations during maintenance of anesthesia enables an assessment of the response to small challenges of opioid analgesics. Controlling respiration precludes such an evaluation. Regarding C and D: In animal models, exposure to intermittent hypoxia during development affects the opioid system, increasing the density of µ-opioid receptors in the respiratory-related areas of the brainstem. This may represent an adaptive response to the effects of recurrent intermittent hypoxia that allows µ-receptor-mediated opioid respiratory effects to predominate. For children with severe OSA, the severity of the nocturnal oxygen desaturation correlates with the sensitivity to exogenously administered opioids. Perioperative opioid use in children with severe OSA results in smaller-than-expected doses of opioids possibly producing exaggerated respiratory depression.

130. Kid with OSA (barely recall): a) Codeine is recommened analgesic b) Should continue with spontaneous respiration c) Sensitive to opioids d) The more they desat at night, the higher their sensitivity to opioids

ANSWER: C (Consensus C) Reference: Cote Ch 15 Pg 343 Management of a tet spell requires urgent intervention. Early and aggressive use of a vasoconstrictor is essential. Phenylephrine should be premixed and in a syringe for immediate use in a concentration of 100 mcg/mL. The initial dose is 0.5 - 1 mcg/kg, which is repeated and doubled in 1-min intervals until a satisfactory response is observed. Preterm infants may require as much as 30 mcg/kg. Management may require any of the following: 100% O2 Hyperventilation IV fluid bolus Sedation or analgesia (eg fentanyl, morphine) Sodium bicarbonate Vasoconstriction o Norepi 0.5 mcg/kg bolus, then rate of 0.01 to 0.2 mcg/kg/min o Phenyl 0.5 mcg/kg bolus and doubled @ 1 min intervals until a satisfactory response is achieved; this is followed by an infusion at 1-5 mcg/kg/min (larger boluses may be required in small preterm infants) -blockers to relax infundibular spasm and reduce the HR o Propranolol 0.1- to 0.3-mg/kg bolus

131. Patient with TOF, becomes hypotensive, tachycardic, and desats. Best treatment a) Ephedrine b) Epinephrine c) Phenylephrine d) Norepinephrine

ANSWER: A (Consensus...not really a consensus, most answers either saying "A + C" or "Except b") References: Barash Ch 41 Pg 1179, Figure from Cote Ch 16 Pg 355 Background: The fetal circulation is characterized by a parallel system in which both ventricles pump most of their output into the systemic circulation. Less than 10% of the combined cardiac output goes through the fetal pulmonary circulation via three main shunts (placenta, foramen ovale, ductus arteriosus). The placenta provides oxygenated blood into the ductus venosus, the IVC, and then into the RA. Regarding A (true) and B (false): In the RA, the majority of oxygenated blood primarily flows through the foramen ovale into the LA, bypassing the RV and pulmonary vascular bed. Some blood from the RA does flow through the RV and into the main PA. Regarding C (false-ish): Some blood in the pulmonary artery does flow through the pulmonary circulation and then into the left atrium, but the majority of flow goes through the ductus arteriosus into the descending aorta. I said this was false based on the technicality of C stating that "some" blood goes through the PDA rather than the "majority" as described in Barash.

133. Fetal circulation a) Blood from RA goes to LA through PFO b) Most blood goes from RA to RV c) Some blood goes to the aorta through PDA

ANSWER: A (Consensus A) Reference: Barash Ch 48 Pg 1387 Excellent candidates for topical analgesia: ♣ Fully anticoagulated patients ♣ Monocular patients who are spared the trauma of prolonged local anesthetic-induced postoperative amaurosis (somewhat Option A) Potential disadvantages: ♣ Eye movement during surgery (Option C) ♣ Patient anxiety or discomfort (Option B) ♣ Allergic reactions (Option D) Patient selection: ♣ Restrict to individuals who are alert and able to follow instructions, and who can control their eye movements ♣ Contraindications/Inappropriate candidates: o Demented o Photophobic o Cannot communicate o Inflamed eye o May require significant iris manipulation or large scleral incisions (eg dense cataracts, small pupils)

134. Disadvantages of topical eye anesthesia, all except a) Delayed recovery of vision b) Patient anxiety c) Eye movement d) allergy

ANSWER: D (Consensus D) Reference: question likely specifically based on Miller Ch 39 Pg 1180 under "Posterior Pituitary Hormone Excess and Deficiency" Background: Secretion of vasopressin or antidiuretic hormone (ADH) is enhanced by increased serum osmolality or the presence of hypotension. Inappropriate secretion of vasopressin, without relation to serum osmolality, results in hyponatremia and fluid retention. Regarding A: (Barash Ch 14 Pg 344) As per table, SIADH is hyponatremia with low plasma osmolality. It is euvolemic hyponatremia. (Miller) SIADH should be suspected in any patient with hyponatremia who excretes urine that is hypertonic relative to plasma. Lab findings that support the diagnosis: 1) Urinary sodium > 20 mEq/L 2) Low serum levels of BUN, Cr, uric acid, albumin 3) Serum sodium < 130 mEq/L 4) Plasma osmolality < 270 mOsm/L 5) Urine hypertonic relative to plasma Regarding B: (Miller) Nothing to indicate this is true directly. The closest are probably these lines: "Despite the common impression that SIADH is frequently seen in older patients in the postoperative period, studies have shown that the patient's age and the type of anesthetic used have no bearing on the postoperative development of SIADH. It is not unusual to see several patients in the neurosurgical ICU suffering from this syndrome." If this question was asking about hyponatremia, but not specifically SIADH, it depends on your definition of substantial...but Barash (Ch 14 Pg 343) states "at least 4% of postoperative patients develop plasma [Na+] < 130 mEq/L. Regarding C: (Miller) 10-20% of long-distance and marathon runners have SIADH with increased vasopressin secretion. Regarding D (false): (Miller) SIADH can result from the following: ♣ Variety of CNS lesions ♣ Drugs: nicotine, narcotics, chlorpropamide, clofibrate, vincristine, vinblastine, cyclophosphamide ♣ Pulmonary infections ♣ Hypothyroidism ♣ Adrenal insufficiency ♣ Ectopic production from tumours

136. All are true for SIADH, EXCEPT a) Serum Na and osmolality are decreased b) It occurs in a substantial number of patients post-op c) Occurs in 3% (?) of long distance runners d) Chlorpropamide is useful for treatment

Regarding A (false): Although renal recovery is anticipated, 35% of patients with pretransplant HRS will continue to require support in the immediate postoperative period. No recovery if no transplant. Regarding B (true): Profound renal vasoconstriction is the proximate cause of HRS. Regarding C (false): Therapy aimed at directly increasing renal perfusion by the use of prostaglandins, dopamine agonists, or endothelin antagonists has not proved successful. More effective has been therapy targeting the underlying pathology that leads to the renal vasoconstrictive response (ie reduction of portal hypertension and/or splanchnic vasodilation). Regarding D (false): HRS accounts for only about 23% of cases of AKI in hospitalized cirrhotic patients. In cirrhotic patients with ascites, the incidence of HRS is 18% at 1 year and 39% at 5 years. Therefore patients with ascites are more prone to HRS.

137. Which of the following describes hepatorenal syndrome? a) It is irreversible. b) Its pathogenesis is related to renal vasoconstriction c) NSAIDs reduce risk d) It is more likely to occur in a cirrhotic without ascites as opposed to one with ascites.

ANSWER: B (Consensus mainly B, one school picked C and one D) Reference: Miller Ch 45 Pg 1365-6 Regarding B: In general, unintended arterial puncture is the most common acute mechanical complication, with incidence ranging from 1.9% to 15%.... Pneumothorax is the most common complication of subclavian vein cannulation, although unintended arterial puncture may actually be more frequent. They describe a study of subclavian venous cannulation with 1.5% incidence of pneumothorax and 3.7% incidence of arterial puncture. Regarding C: The most important life-threatening vascular complication of central venous catheterization is cardiac tamponade resulting from perforation of the intrapericardial SVC, RA, or RV. This injury was the second-most common complication related to central catheters reported in the Closed Claims Project in 2004. There was often a delayed presentation (1-5 days), therefore more likely related to catheter maintenance and use than to the vascular access procedure itself. Early reports of catheter-related cardiovascular perforation suggested that peripheral catheters may present an unusually high risk for this because arm abduction may cause the catheter tip to advance into a dangerous location within the heart. Regarding D: More than 15% of patients experience adverse event related to central venous cannulation.

138. CVP Complications a) a) ?? b) b) Most common complication with subclavian insertion is arterial puncture c) c) Less risk of cardiac tamponade with peripherally inserted central line d) d) Overall complication rate around 5%

ANSWER: B (Consensus mainly B, 2 schools picked A) Reference: Barash Ch 37 (Anesthesia for Thoracic Surgery) Ch 1054-6 Regarding A and B: Desflurane inhibits HPV, with an ED50 of 1.6 MAC. Propofol infused in doses of 6-12 mg/kg/h does NOT abolish HPV during OLV in humans. Propofol infusion in combination with remifentanil is probably the technique of choice for producing a stable OLV with no effect on HPV.... It is generally believed that inhaled agents inhibit HPV, whereas IV drugs do not have this effect. Regarding C: Direct inhibitors of HPV include infection, vasodilator drugs (nitroglycerine and nitroprusside), hypocarbia, and metabolic alkalemia. Regarding D: Factors associated with an increase in pulmonary artery pressure antagonize the effect of increased resistance caused by HPV and result in increased flow to the hypoxic region. Such indirect inhibitors of HPV include mitral stenosis, volume overload, thromboembolism, hypothermia, vasoconstrictor drugs, and a large hypoxic lung segment.

139. All inhibit HPV except: a) Inhalational anesthetics b) IV anesthetics c) Nitroglycerine d) Dopamine

Consensus D Answer D Beta 2 • Selective B2 adrenergic agonist effects of epinephrine are speculated to reflect activation of the sodium-potassium pump in skeletal muscles, leading to transfer of potassium ions into cells

14. Epinephrine effect causing hypokalemia through what receptor? a) Alpha 1 b) Alpha 2 c) Beta 1 d) Beta 2

ANSWER: B (Consensus mainly B; one school picked B+D) Reference: Barash Ch 39 (Anesthesia for Vascular Surgery) Pg 1129 Background: The decision on which treatment modality is best for a large AAA usually depends on 3 variables: AAA size, AAA morphology, and patient perioperative risk. Regarding A (true): The EVAR-1, DREAM, and OVER trials all showed 30-day mortality to be significantly lower with EVAR than OAR (1.8%, 1.2%, and 0.5% vs 4.3%, 4.6%, and 3%, respectively). Regarding B (probably false): Two years after randomization (OVER trial), there was no significant difference in mortality, major morbidity, procedure failure, secondary therapeutic procedures, aneurysm-related hospitalizations, health-related quality of life, or erectile dysfunction. No discussion specifically on daily living at 1 year. Regarding C (true): The OVER trial showed that EVAR had a statistically significant reduction in procedure time, blood loss, transfusion requirement, duration of mechanical ventilation, ICU stay (1 vs 4 days), and hospital stay (3 vs 7 days), Regarding D (true): EVAR has lower perioperative risk than OAR, but similar 2-year mortality. EVAR also has a higher incidence of leak, thus necessitating a lifetime of surveillance.

140. EVAR compared to open AAA decreases all except a) 30 day mortality b) Daily living at 1 year c) Length of stay in hospital d) Post op complications

- The Ramsay Sedation Scale Can be used to monitor depth of sedation - Not used for pain - Can be used on ventilated patients, provided not paralyzed - IS found on CAS guidelines, appendix 6

141. Ramsay Scale a) Used for depth of anesthesia b) Used for pain c) Can't be used in ventilated patients d) Referenced by CAS on the website

Consensus B Answer B PaCO2 40mmHg and pH 7.4 (7.35) • Normal PaCO2 of newborn = 30-35mmHg, pH 7.3-7.4 • Regarding A blood sugar 2.7 • Normal • Studies have defined values for glucose levels that should be cause of concern: plasma glucose levels less than 35 mg/dL (<2 mmol/L) in the first 3 hours of life; less than 40 mg/dL (2.2 mmol/L) between 3 and 24 hours; and less than 45 mg/dL(2.5mmol/L) after 24 hours. • Others have defined hypoglycemia in full term infants as a plasma glucose concentration of less than 30 mg/dL (1.6 mmol/L) in the first day of life or less than 40 mg/dL (2.2 mmol/L) in the second day of life • Cote pg 18 Regarding C systolic blood pressure of 65mmHg • Normal BP 65-85 mmHg Regarding D minute ventilation 700mL • The tidal volume varies between 10-30mL, the breathing frequency ranges from 30-60 breaths per minute and minute ventilation exceeds 500mL (Barash pg 1170) [Note different than values from cote table above)

15. A child of 3.5kg born by c-section and has now 5 minutes of life, which of these findings is unusual: a) blood sugar 2,7 (2.5) b) PaCO2 40 mmHg and PH 7.4 (7.35) c) Systolic blood pressure of 65 mmHg d) Minute ventilation of 700 ml/min

Consensus A Answer A Dilated Cardiomyopathy • Cardiomyopathy occurs in nearly all patients with DMD. Dystrophin is present in cardiac muscle cells, and the degeneration of cardiomyocytes results in fibrosis. The fibrosis initially affects the posterobasal segment of the left ventricle, which leads to increased myocardial wall stress and a progressive decrease in left ventricular systolic function that eventually results in ventricular dilatation and dysfunction. Theses changes are evident as characteristic electrocardiographic abnormalities including tall R waves over V1; deep and narrow Q waves over I, V5 and V6; sinus tachycardia and right axis deviation. Initially the echo will show areas of regional wall motion abnormalities at the sites of fibrosis, and eventually, ventricular dysfunction will be apparent. Fibrosis of the posterior papillary muscle can result in significant mitral regurgitation. As a consequence of the structural changes, patients with DMD are highly susceptible to a range of arrhythmias (Miller chapter 42 pg 1273) • Degeneration of cardiac muscle invariably accompanies this muscular dystrophy. Characteristically, the electrocardiogram reveals tall R waves in V1, deep Q waves in the limb leads, a short PR interval, and sinus tachycardia. Mitral regurgitation may occur as a result of papillary muscle dysfunction or decreased myocardial contractility. (CoE chapter 21 pg 445)

16. 25 yo male with Duchene's Muscular Dystrophy undergoing laparoscopic cholecystectomy, what is the most likely cardiac anomaly? a) Dilated Cardiomyopathy b) Ventricular Septal Defect c) Right Ventricular arrythmogenic dysplasia d) AV Conduction delay

Consensus A Answer A Quinke cutting needle - True • PDPH decreases with the use of small diameter spinal needles with noncutting tips (Barash Page 926) Regarding A Age > 60 PDPH decreases with increasing age (Barash Page 926) Regarding C Male • Some studies have suggested that women have a higher risk for PDPH Regarding small gauge needle • PDPH decreases with use of small diameter spinal needles with noncutting tips (Barash Page 926) Summary from Barash RFs that decrease risk • Increasing age • Small diameter spinal needle with noncutting tips • Inserting cutting needles with the bevel aligned parallel to the long axis of the meninges o Parallel insertion spreads dural fibres whereas perpendicular insertion cuts fibres resulting in larger meningeal hole • Male ("however if age differences are accounted for there does not appear to be a gender difference in the incidence of PDPH) • Saline

17. What increases the risk of headache with CSF leak after subarachnoid block? a) Quinke Cutting Needle b) Age>60 c) Male d) Small Gauge Needle

Consensus Answer C - 13 schools Answer B - 1 school Alberta Answer B & C - McGill Answer C Vagal nerve supplies sensation to the vocal cords • Sensory innervation of the larynx down to the vocal cords is supplied by internal laryngeal nerve branches of the superior laryngeal nerves (internal laryngeal nerve), theses in turn being branches of the vagus (Miller Chapter 85 pg 2525) Summarized • Glossopharyngeal o Via Pharyngeal nerves - pharyngeal mucosa o Via Tonsillar nerves - mucosa overlying palatine tonsil and soft palate, posterior third of tongue • Vagus o Via superior and recurrent laryngeal nerves - Mucosa of the airway from the level of the epiglottis to distal airways o Via superior laryngeal nerve internal branch surfaces of epiglottis and to the airway mucosa to the level of the vocal cords My summary for sensory innervation (based off M5 board review question I did) ♣ Anterior tongue - trigeminal, mandibular branch v3 ♣ Posterior tongue - glossopharyngeal ♣ Soft palate glossopharyngeal ♣ Oropharynx glossopharyngeal ♣ Hypopharynx (oropharynx below level of epiglottis) internal branch of superior laryngeal nerve CN X ♣ Vocal cords - both internal branch of superior laryngeal nerve and recurrent laryngeal (both CN X) ♣ Larynx (area below vocal cords but above trachea) recurrent laryngeal ♣ Trachea - recurrent laryngeal Toronto Notes summary • Glossopharyngeal from posterior 1/3 tongue to anterior epiglottis • Internal branch of superior laryngeal nerve from epiglottis to cords • Recurrent laryngeal nerve from cords to trachea Regarding B Glossopharyngeal nerve supplies sensation supraglotically • This is technically also true as demonstrated by figure 27-1 above • This answer choice may have been misremembered?? The word supraglottic could have meant subglottic • Note question in 2012 had this as an answer choice "Glossopharyngeal nerve is responsible for subglottic sensory function" • Subglottic innervation is RLN Regarding A All motor to the larynx supplied by the RLN - False • The two recurrent laryngeal nerves provide motor innervation to all the intrinsic muscles of the larynx except • Cricothyroid and inferior pharyngeal constrictor muscles are supplied by the external branches of the two superior laryngeal nerves (External laryngeal nerve) Regarding D Glossopharyngeal nerve supplies motor infraglotically - False • See answer to A

18. Regarding Airway innervation which one is true? a) All motor to the larynx supplied by the RLN. b) Glossopharygeal nerve supplies sensation supraglottically c) Vagal nerve supplies sensation to the vocal cords. d) Glossopharygeal nerve supplies motor infraglottically.

Answer A • The question describes an individual with chronic alcohol abuse and likely suffers from delirium tremens • Benzodiazepines are the first line treatment for delirium tremens in both Miller and CoE hence the reason why answer A was chosen o Withdrawal symptoms can be supressed by: ♣ Resumption of alcohol ♣ Benzodiazepines ♣ B-Blockers ♣ Alpha 2 agonists ♣ B blocker is added if tachycardia is present. ♣ (Co-Existing Chapter 25 pg 544) o Treatment of delirium tremens must be aggressive administration of diazepam (5 to 10 mg IV every 5 minutes) or another benzodiazepine until the patient becomes sedated but remains awake. Administration of B blockers such as propranolol and esmolol is useful to supress manifestations of sympathetic hyperactivity (CoE pg 544) o Miller chapter 39 pg 1199 - DT treated with benzodiazepines Regarding B Propofol • I gather other schools have chosen this as a possibility as the question describes a cirrhotic patient. Midazolam may be prolonged in cirrhotic patietns as opposed to propofol which could be used for sedation o Methohexital and propofol also have an elimination kinetic profile in cirrhotic individuals that is similar to that observed in normal patients, both after single bolus administration of propofol and after propofol infusion an effect that may result from extrahepatic metabolism of propofol. However, mean clinical recovery times may be longer after discontinuation of propofol infusions in cirrhotic patients. The reduced clearance of midazolam in patients with end stage liver disease produced prolonged elongation half lifes (Miller Chapter 73 pg 2248) Regarding C Risperdal • This has not been recommended for DT Regarding D Dexmedetomidine • According to CoE Alpha 2 agonists may be used for withdrawal symptoms although it is not the first line treatment. • It is also metabolized in the liver Dexmedetomidine, an alpha 2 adrenergic agonist with sedative an analgesic properties is primarily metabolized in the liver with minimal renal clearance

2. We ask your help at the cardiac intensive care unit for a patient who just had a PCI on his right coronary artery. He is known for chronic alcohol abuse, with a cirrhosis graded as child C. He is agitated. What is the best treatment to use to control is agitation: a) Midazolam b) Propofol c) Risperdal d) Dexmedetomidine

Consensus D Answer D Amiordarone • Two studies listed in Miller which demonstrated improved survival to hospital admission with amiodraone a. ARREST Study - amiodarone in out of hospital resuscitation of refractory sustained ventricular tachycardia (ARREST) reported results of randomized prospective study in which 300 mg IV amiodarone or placebo was administered to patients in out of hospital cardiac arrest with persistent VF after three defibrillation attempts and 1 mg IV epinephrine. i. The study demonstrated significant increases in survival to hospital admission in the amiodarone treated group but was unable to demonstrate any difference in hospital discharge rates. b. ALIVE study - improvement produced by amiodarone only in survival to hospital admission, not to discharge, when compared with lidocaine or placebo. (Miller Chapter 108 pg 3203)

22. Which anti-arrhythmic agent has evidence for improved survival to hospital admission? a) Metoprolol b) Procainamide c) Lidocaine d) Amiodarone

Answer A From Cousins p.410 of the PDF, placement of neuraxial anesthetic can theoretically transfer pieces of pigment into the subarachnoid space which could cause inflammation and granulomatous tissue formation. Use of an introducer needle does not abolish this risk. Regarding B, C, and D - If possible, use an interspace which does not have ink covering it. If the tattoo must be pierced, it has been suggested to make a superficial skin incision prior to placing the introducer needle, and to let a few drops of CSF drip from the needle prior to placing the anesthetic.

29. In regards to doing a neuraxial technique on a patient with a tattoo all improve safety except (or which of the following is correct)? a) The introducer needle always prevents coring of epidermal ink b) When using a spinal needle making a small cut in the tattoo improves safety c) Once Spinal needle in the subarachnoid space let a few drops of CSF leak back before injecting medication. d) Avoid placing the spinal needle in the tattoo altogether

Answer A Laser thermal effects are similar to those from other surgical sources, except that they can be more finely focused with high-energy density and delivered via fiberoptic strands. Regarding B and D, CO2 lasers emit far-infrared frequencies that are unable to pass through any clear glass or plastic lenses. Regular eye-glasses, fitted with side shields, may be sufficient protection, but contact lenses are not. All other lasers emit energy that requires specific colored lenses. Regarding C, UV and far-infrared light is absorbed by the anterior structures. Infrared energy from a CO2 laser causes corneal and lens injury. Miller p.2601, 2605

31. In regards to CO2 laser which is True: a) The laser is transmitted down fiberoptic cable b) The C02 will pass through a plate glass c) C02 scattered array will burn the retina d) plastic glasses provide insufficient protection

Answer C For patients with hypomagnesemia, ventricular dysrhythmias should be anticipated and treated as necessary. Regarding A, hypomagnesemia can aggravate digoxin toxicity and CHF. Regarding B, hypokalemia and/or hypocalcemia that had been refractory to supplementation will respond after correction of hypomagnesemia. From Barash, it is associated with hypokalemia, hyponatremia, hypophosphatemia, and hypocalcemia. Regarding D, loss of deep tendon reflexes is associated with HYPERmagnesemia, not hypomagnesemia. Hypotension, nausea, lethargy, and facial flushing are also signs of hypermagnesemia. Coexisting p.368-369 Barash p.357

32. Which of the following is associated with hypomagnesemia? a) Resistance to digoxin toxicity b) Uncommon to be associated with hypokalemia c) Associated with ventricular arrhythmia d) Loss of deep tendon reflexes (not sure??)

Answer C Stand beside the patient and identify the C6 vertebra (chaissagnac's tubercle) by palpation. Next place the index and 3rd digits between carotid laterally and trachea medially. Needle is inserted until it contacts the C6 transverse process, then withdrawn 1-2mm and local anesthetic injected.

35. For stellate ganglion block, using the most common anterior approach, the transverse process is contacted where? a) C4 medial to carotid b) C4 lateral to carotid c) C6 medial to carotid d) C6 lateral to carotid

Answer C Increased risk of postop myasthenic crisis and ventilatory support with the following conditions: 1)duration of disease >6 years; 2)presence of previous respiratory problems or coexisting lung disease; 3)pyridostigmine dose >750mg/day; 4)FVC <2.9L Regarding A, after surgery mechanism for improved symptoms is uncertain, though antibody titers usually decrease, however, full benefit of thymectomy is often delayed for months after surgery. Regarding B and D, the ED95 of sux is 2.6X HIGHER than a normal patient due to the decreased numbers of functional Ach receptors. Hence they are resistant to depolarizers. They are sensitive to nondepolarizers.

36. Myasthenia gravis patient for sternotomy and thymectomy under GA. Which is true? a) After surgery, immediate drop in antibody titers b) ED95 of succinylcholine is 2.5 times lower than normal patients c) Pyridostigmine >750mg/day associated with prolonged post-op ventilation d) They are sensitive to depolarizing muscle relaxants

Answer B Regarding regional vs general anesthesia for orthopedic procedures, Miller states regional anesthesia may reduce incidence of complications such as DVT, PE, blood loss, respiratory complications, and death. Miller p.2394; Barash p.1449 Regarding A, the highest risk of DVT occurs postpartum. It peaks in the first week postpartum and declines from then onwards. Chestnut p.920 Regarding C, I couldn't find anything in our textbooks specifically addressing this. Miller p.2165 discusses the fact that neurosurgical patients are high risk for DVT and as such measures should be taken to address this. Multiple articles talking about use of UFH or LMWH in neurosurgical patients, and anecdotally we don't hold prophylaxis unless there is a concern regarding bleeding. Regarding D, LMWH is only partially reversed with protamine, as opposed to UFH which can be completely reversed. Miller p.1873-1874.

37. Regarding DVT prophylaxis, which is true a) Parturients are at increased risk of DVT compared to post-partum patients b) Neuraxial lowers rates of DVT in patients undergoing hip surgery c) Thromboprophylaxis should be avoided in neurosurgery patients d) LMWH is better than UFH because of easier reversal with protamine

Answer C Taken from Kaplan figure 19-13 and Miller figure 67-28 which show for aortic stenosis a slight increase in ESV, and significant increase in pressure. Chronic outflow obstruction results in chronic pressure overload which stimulates LVH and thus the peak pressure the LV is able to generate is much higher, and is required to be higher to overcome the outflow obstruction. In simple terms, an increase in preload makes the pressure-volume diagram wider whereas an increase in afterload makes it taller. Regarding A, a VSD results in 2 paths of ejection for the stroke volume. Assuming a normal LVOT there should be no significant increase in pressure from this aspect, and assuming a normal PVR and RV compliance this will provide a low-resistance outlet, decreasing any pressure generated. Regarding B, ischemic cardiomyopathy results in poor contractility and hence a decrease in pressure generated in systole. It may also be associated with a much more significant change in volume as the heart becomes volume overloaded due to poor EF. Regarding D, diastolic dysfunction means a stiff noncompliant ventricle. This will result in the diastolic limb being steeper, but with no outflow obstruction there will be no significant increase in the systolic pressure component.

38. When looking at the left ventricular pressure volume curve, which of the following is associated with slight change in volume and increased pressure at the beginning of systole? a) VSD b) Ischemic cardiomyopathy c) Aortic stenosis d) diastolic dysfunction

Answer C From Barash, BIS readings are affected by a number of variables which raise questions regarding their validity in children. 1)BIS readings are affected by anesthetic administered (at equi-MAC values, BIS measurements are 50% higher with halothane than with sevoflurane) 2)Variability of BIS measurements among children prevents precise interpretation 3)age directly affects BIS readings - in children <5yrs they are much less reliable, likely from maturational changes in EEG waveforms from birth to school age, not incorporated into BIS algorithm 4)BIS readings decrease as sevo concentration increases, until 3%, after which BIS readings paradoxically increase 5)BIS cannot accurately measure changes when ketamine or nitrous are used Barash p.1239 Regarding A, Cote states that incidence of awareness in pediatric anesthesia is 0.2-1.2%, but that children who do experience awareness do not develop long-term sequelae. Cote p.103 Regarding B, Barash cites an incidence of 1%, which is 3-10X that reported in adults. Since 4-8X falls within this distribution I took this as being a correct statement. Barash p.1236 Regarding D, nothing explicit that states adding nitrous decreases awareness, however, it is an amnestic agent, and in addition, Barash does state that his only indications to use BIS are those who cannot tolerate GA due to hemodynamic instability, those in whom nitrous is not used, and those who require TIVA. This infers that awareness is decreased with nitrous usage. Barash p.1239

43. Regarding awareness in children, except? a) Most children are not distressed b) Incidence is 4-8 times that of adults c) BIS is an effective tool to prevent it d) Adding N2O will decrease awareness

Answer B Thyroid storm most often occurs in the postop period in untreated or inadequately treated hyperthyroid patients after emergency surgery. Regarding A, as established above, this occurs more frequently in untreated or poorly treated patients. Regarding C, beta-blockers should be titrated to HR <90. Dex 2mg q6h or cortisol 100-200mg q8h can be used to decrease hormone release and conversion of T4 to T3. PTU 200-400mg q8h may be administered NG, PO or PR. If circulatory shock is present, vasopressor therapy should be inititated. Regarding D, thyroid storm manifests as extreme anxiety, fever, tachycardia, cardiovascular instability, and altered LOC. Additionally, although cardiac failure rarely occurs, a thyrotoxic cardiomyopathy has also been described. As such, it can cause hypotension although this is not the most common sign.

45. Regarding thyrotoxicosis, which is false? a) More frequent in poorly controlled hyperthyroidism b) Usually occurs intraop c) Should be treated with beta-blockers d) Can cause hypotension

Answer A or D Regarding A, interaction of meperidine with MAOI's is the most classic for serotonin syndrome according to Barash. Regarding D, Barash additionally mentions methylene blue as being a potent reversible MAOI, and states that it has been associated with serotonin syndrome. Venlafaxine is a SNRI which can also contribute to elevated 5HT levels. Regarding B, the phenylpiperidine series of opioids (fentanyl and its congeners, meperidine, tramadol, and methadone) have been associated with serotonin syndrome. Regarding C, cyproheptadine is a 5HT-2A antagonist used to treat serotonin syndrome, but is is for oral use only.

49. In a patient with serotonin syndrome a) Avoid meperidine with MAOIs b) Fentanyl is safe to give to a patient on MAOI c) Treatment with IV cyproheptadine d) Methylene blue given to a patient on Venlafexine can cause serotonin syndrome

Consensus B Answer B hydrostatic error because the cuff on the extremity is lower than the RA/ reading will be affected o I believe this answer choice is attempting to test one's understanding the differences in invasive and non invasive blood pressure measurements and changes in position o A hydrostatic error will result in different NIBP measurements between extremities while invasive BP measurements stay the same o For example in the lateral decubitus position o Having the NIBP on the lower (dependent) extremity will result in a higher BP. A NIBP on the upper (nondependent) extremity will result in a lower BP o Invasive blood pressure - no change in BP between extremities ♣ For proper interpretation of blood pressure measurements from a patient in the lateral decubitus position, it is helpful to differentiate zeroing and leveling pressure transducers and to appreciate the differences between non invasive and invasive blood pressure measurements, in this position although the aortic root remains stationary, one arm is necessarily higher than the other. However, as long as the pressure transducer remains fixed at the level of the heart, the location of the arms or in which the the vessel of the catheter resides has no difference the the measured arterial pressure. On the other hand, non-invasive cuff blood pressure measurements will be different in the two arms - higher in the dependent (down) arm and lower in the nondependent (up) arm. (Miller chapter 45 pg 1355) o Regarding The cuff is too small which underestimates the BP - False • Small cuff - overestimates • Large cuff - underestimates/ provides acceptable results o Although a too large cuff often provides acceptable results, the use of a cuff that is too small usually results in too high readings. o A well fitted cuff has a bladder that extends to 40% arm circumference and 80% length of upper arm Regarding C the systolic pressure by oscillometry is better than MAP - False • Systolic pressures are calculated, MAPs are measured o Most NIBP devices are based on oscillometry... in this method small changes in cuff pressure with arterial pulsation during cuff deflation are used to estimate MAP. o SBP and diastolic pressures are calculated to proprietary algorithms that vary by manufacturer; they are less reliable than the values for MAP. Regarding D systolic pressure with NIBP is higher than arterial line - False • Systolic pressure with NIBP is lower o Clinical studies comparing NIBP with direct arterial pressure measurements also reflect the problematic nature of NIBP monitoring. o Oscillometric methods often underestimate systolic and overestimate diastolic measurements, significantly underestimating pulse pressure calculations

5. Case under GA, you suspected that the NIBP is not working correctly, which is true a) the cuff is too small, which understimate the BP b) Hydrostatic error because the cuff on the extremity is lower than the RA / reading will be affected by elevation over apex c) the Sysolic pressure by oscillometry is better than MAP d) systolic pressure with NIBP is higher than arterial line

Answer C The Glenn procedure is a palliative procedure which bypasses the right side of the heart and results in passive return of blood to the pulmonary circulation. As a palliative procedure it requires endocarditis prophylaxis. Barash table 42-9 Regarding A, the VSD was repaired 9 months ago, therefore unless there is a residual defect it does not require prophylaxis. Regarding B, assuming the TOF was repaired and no residual defects there should be no need for prophylaxis. Regarding D, a PDA is not a cyanotic lesion and does not need prophylaxis.

51. 3y-o female, which one would be an indication for infective endocarditis prophylaxis? a) VSD reparired 9 months ago b) TOF repaired at the age of 6 month c) Tricuspid atresia with a Glenn at age of 6 month d) Unrepaired PDA

Answer D Barash states that 70 days is the cutoff for octafluorocyclobutane. As such, 90 days is the only option that would be safe. Of note, cutoffs are 5 days for air injection, 10 days for sulfur hexafluoride. Nitrous should be discontinued 15mins before any of these gases are injected. No mention of octafluorocyclobutane in Miller. Barash p.1381

52. How long do you need to avoid N2O after using octafluorocyclobutane a) 8days b) 28 days c) 60 days d) 90 days

Answer C Whether NIV should be used in patients with acute asthma, upper airway obstruction, and trauma is not yet clear. Established indications for NIPPV are COPD exacerbations, acute cardiogenic pulmonary edema, postop respiratory failure, and respiratory failure in immunocompromised patients (solid organ transplants and bone marrow transplants).

53. Not indicated for NIPPV a) Immunocompromised patient b) COPD c) Asthma d) Cardiogenic pulmonary edema

Answer D From Miller p.2403 regarding pulmonary function and complications in scoliosis, thoracic scoliosis results in narrowed chest cavity producing a decrease in chest wall compliance and restrictive lung disease. Cobb angles >65 degrees usually cause significant decreases in lung volumes. Although exercise tolerance is an important determinant of the effects of severity of the curve on respiratory function, formal PFTs should be obtained before surgery. This info guides decisions regarding extent of surgery permitted at one time, and requirement for postop ventilator support. A vital capacity of <40% of normal is predictive for requiring postop ventilation. The major abnormality on ABGs is hypoxemia secondary to alveolar hypoventilation. From Coexisting p.452, spinal curvature of >40 degrees is considered severe and is likely to be associated with physiologic derangements in cardiac and pulmonary function. Restrictive lung disease and pulmonary HTN progressing to cor pulmonale are the principal causes of death in patients with kyphoscoliosis. From Barash p.1443, the severity of scoliosis is defined by the Cobb angle. Surgical correction is performed for Cobb angles >50 degrees with the intent of halting, not reversing, progression of cardiac and pulmonary dysfunction. In patients with untreated scoliosis, respiratory failure and death usually occur by 45 years of age. Vital capacity appears to be a reliable prognostic indicator for perioperative respiratory reserve. Postop ventilation will likely be required for those with vital capacity <40%. Although the long term goal is to halt respiratory dysfunction, respiratory status will acutely decline for 7-10 days postop. Regarding A, number of vertebrae in the curve, and number of vertebrae requiring fusion are a factor in predicting surgical blood loss, but no mention specifically of factoring into pulmonary function. Regarding B, most curves are right-sided. Left-sided curves are more likely to be associated with other thoracic abnormalities, but no mention specifically of being a specific factor for bad pulmonary function. Regarding C, no mention of rigidity of the curve being a factor for bad pulmonary function.

54. Which of the following is a predictor of respiratory compromise in scoliosis a) Number of vertebrae involved b) Morphology c) Rigidity d) Vital capacity

Answer E From Cote p.378, PGE1 infusions are indicated to maintain a patent ductus arteriosus in infants with duct-dependent CHD (pulmonary atresia, tricuspid atresia, severe TOF, HLHS, critical AS, interrupted aortic arch. It is infused at a rate beginning at 0.05mcg/kg/min and may be increased to 0.1mcg/kg/min. PGE1 infusions are associated with CNS irritability, not depression. Regarding A, Chestnut table 38-3 lists side effects of misoprostol (a synthetic PGE1 analog) as being fever, chills, N/V, and diarrhea. Regarding B and D, from Cote p.378, adverse effects of PGE1 infusion include systemic hypotension, apnea, increased risk of infection, leukocytosis, gastric outlet obstruction, and CNS irritability. Regarding C, Cote p.623 discusses PGE1 infusions in the context of lung transplants as a way to prophylactically reduce PVR or to treat PHTN. Additionally p.728 discusses the fact that PGE1 reduces PVR but often at the expense of causing systemic vasodilation.

56. All of the following statements EXCEPT ONE are true about the potential side effects of PGE1 intravenous infusion. Identify the exception. a) increased body temperature b) apnea c) decreased pulmonary vascular resistance d) systemic vasodilation e) CNS depression

ANSWER: D (consensus) WHY: Although the Barash figure below shows the sensory distribution of the tibial nerve reaching up to the knee, its sensory distribution is largely to the plantar surface of the foot. Articular branches from the tibial nerve would have come from the sciatic component prior to it dividing. Barash Fig 35-14 Barash ch. 50 p. 1450: continuous peripheral nerve techniques that block the lumbar plexus (fascia iliaca, femoral, psoas compartment blocks), with or without sciatic nerve blockade, can be used with success for patients having total joint replacement. Innervation of the knee joint is derived from (i) femoral nerve via the branch to vastus medialis (anterior aspect of the joint capsule); (ii) sciatic nerve via genicular branches of both tibial and common peroneal components (posterior aspect of the joint capsule and all of the intra-articular structures); and (iii) obturator nerve by a branch from its posterior division that accompanies the femoral artery through adductor magnus into the popliteal fossa. Cutaneous innervation of the anterior aspect of the knee is supplied by the femoral nerve. The obturator nerve supplies the skin on the medial aspect of the knee in less than 40% of people. NYSORA: The addition of the sciatic nerve block to the femoral nerve block did not provide additional analgesic benefits. In contrast, Ganapathy et al and others found that blocking S1 as well as the femoral, lateral femoral cutaneous, and obturator nerve was beneficial in enhancing analgesia after TKR.

57. What nerve do you need to block for knee arthrotomy, except? a) Obturator b) Lateral femoral cutaneous c) Sciatic d) Posterior tibial

ANSWER: A (consensus) Metoprolol is a relatively selective β-blocking drug with β-blocking effects at moderate and high doses. It has neither intrinsic sympathomimetic activity nor membrane-stabilizing activity. It is mostly metabolized in the liver, with only 40% of the drug remaining after first-pass metabolism, and only about 5% excreted unchanged in the urine. The elimination half-life is 3.5 hours. Propranolol is extensively bound (90-95%) to plasma proteins. Protein binding of metoprolol is low; it is estimated to account for about 10% of the drug. None of metoprolol's hepatic metabolites have been identified as active. (Previous years' questions state "more lipophilic than propranolol" which is also false - see Barash table)

58. Metoprolol a) oral bioavailability is less thant 50% b) more protein bound than propranolol c) half life is 10 hours d) has active metabolites

ANSWER: C (consensus) WHY: Intramuscular atropine and succ will take longer to work than IV and dose should be succ 4-5 mg/kg IM Management of laryngospasm requires a multifaceted and immediate response. As soon as the diagnosis is suspected, a tight-fitting face mask should be applied to the child's face and 100% oxygen delivered with continuous positive airway pressure (15 to 20 cm H2O dialed into the adjustable pressure limiting valve). Pressures in excess of 20 cm H2O may cause gastric inflation. Note that the reservoir bag should not be squeezed except during the child's inspiratory efforts, lest gas be driven into the stomach. If the triggering event is blood, secretions, or foreign material in the airway, these should be removed immediately. As soon as the offending agent has been expunged, the "jaw thrust" maneuver should be applied. If positive pressure ventilation, 100% oxygen, and jaw thrust maneuver fail to break the laryngospasm, further intervention should be undertaken before desaturation and bradycardia develop. Appropriate treatment would include in the following order: IV or IM atropine (0.02 mg/kg), IV propofol (1 mg/kg), and IV or IM succinylcholine (1 to 2 mg/kg IV or 4 to 5 mg/kg IM).

59. A 4 year old child underwent an eventful hernia repair, develops laryngospasm post-op. What is the least effective measure a) propofol 2mg/kg IV b) atropine 0.02mg/kg IV and succinylcholine 1mg/kg IV c) atropine 0.02mg/kg IM and succinylcholine 1.5mg/kg IM d) jaw thrust, CPAP at 15cm of H2O and FiO2 1.0

Answer B - Below lists the the recommended management of airway fires. Stopping airway gas flow and disconnected the circuit is the second step In management. Regarding answer A pulling out the ETT is the first step however stopping gas flows, removing other flammable materials and pouring saline is recommended first and then restabilising ventilations by mask before reintubating. Miller steps to a/w fire Fire in the airway: 1. Remove the tracheal tube or breathing circuit fire. Hand it off to be extinguished and inspected. 2. Stop airway gas flow (consider disconnecting the circuit). 3. Remove all flammable and burning materials from the airway. 4. Pour saline or water into the patient's airway (to extinguish the fire and cool the tissues). Nonairway fire: 1. Stop airway gas flow. 2. Remove drapes, flammable, or burning material from the patient. 3. Extinguish any burning items. After the airway fire is extinguished: 1. Reestablish ventilation by mask. Avoid supplemental oxygen or N2O, if possible. 2. Examine the ETT to evaluate whether fragments have been left in the airway. Consider bronchoscopy to assess injury and to remove foreign bodies. 3. Assess the patient's status, and devise a plan for ongoing care. After the nonairway fire is extiguished: 1. Assess the patient's status, and devise a plan for ongoing care. 2. If not intubated, assess for smoke inhalation. If the fire is not extinguished on the first attempt, then: 1. Use a CO2 fire extinguisher. 2. If the fire persists, then: a. Activate the alarm system. b. Evacuate the patient, if feasible, and follow institutional protocol. c. Close the door to the OR. d. Shut off the medical gas supply to the OR.

6. During an airway case with a laser suddenly there is a fire in the airway, which of the following is most appropriate to do immediately? a) Pull out the endotracheal tube, and replace. b) Shut off ventilation of gas, and disconnect the circuit. c) Pour saline into the airway to put out the fire. d) Examine the airway with a fiberoptic scope to examine the damage.

ANSWER: C (consensus) WHY: Ketamine in HOCM is not preferred - it increases cardiac rate and contractility (bad) as well as SVR (good) - but is not an unequivocally bad as nodal rhythm, which deprives the heart of atrial kick and preload. Because of the dependence of preload on atrial contraction, control of atrial rate and rhythm is mandatory. If junctional rhythm occurs, these patients may need atrial pacing with a transesophageal pacing probe or a PAC with pacing capability.

60. A patient has HOCM, which of the following will cause the most significant decrease in CO a) trendenlenburg head tilt b) something about increasing SVR that was clearly wrong c) nodal rhythm d) ketamine

ANSWER: B (consensus) WHY: Fires from desiccated absorbant have been described following interaction of sevoflurane with Baralyme (barium hydroxide) not soda lime (sodium hydroxide) and indicator dye is not always accurate. Sevoflurane interacting with desiccated Baralyme®, resulting in fires within the breathing system and severe patient injury. Although other sources of ignition and fire in the breathing system continue to be described, the Baralyme®-sevoflurane problem is somewhat unique in that nothing "unusual" is added to or removed from the breathing system for this to occur. In August 2004, the manufacturer of Baralyme (Allied Healthcare Products) discontinued the sale of this absorbent. Absorption of CO2 by absorbents such as soda lime occurs by a series of chemical reactions; it is not a physical process like soaking water into a sponge. CO2 combines with water to form carbonic acid. Carbonic acid reacts with the hydroxides to form sodium (or potassium) carbonate and water. Calcium hydroxide accepts the carbonate to form calcium carbonate and sodium (or potassium) hydroxide. When the absorbent is fresh, the pH exceeds the critical pH of the indicator dye, and it exists in its colorless form. However, as absorbent becomes exhausted, the pH decreases below 10.3, and ethyl violet changes to its violet form because of alcohol dehydration. This change in color indicates that the absorptive capacity of the material has been consumed. Unfortunately, in some circumstances ethyl violet may not always be a reliable indicator of the functional status of absorbent. For example, prolonged exposure of ethyl violet to fluorescent lights can produce photodeactivation of this dye.

61. Which of the following is true about soda lime a) dessicated absorbants can cause fire with sevo b) requires carbonic acid intermediate c) indicator is an accurate representation of absorbant capacity d) ??

ANSWER: D (consensus) Codeine is transformed to morphine via the enzyme cytochrome P450 2D6, and has an NNT of 16.7. Genetic factors can affect the behavior of these drugs. Approximately 9% of white people do not have the enzyme and do not experience analgesia from codeine.

62. Which medication will patient's derive no analgesic benefit from (in 5-7% [or 7-10%] of population) a) hydromorphone b) meperidine c) morphine d) codeine

ANSWER: C (consensus) WHY: Prospective with historical control is inferior to case-control (=retrospective) Longitudinal studies are classified by the method with which the research subjects are selected. These methods for choosing research subjects can be either prospective or retrospective; these two approaches are also known as cohort (prospective) or case-control (retrospective). Prospective studies are further divided into those in which the investigator performs a deliberate intervention and those in which the investigator merely observes. Studies of deliberate intervention are further subdivided into those with concurrent controls and those with historical controls. Concurrent controls are either a simultaneous parallel control group or a self-control study; historical controls include previous studies and literature reports. Systematic Review: An article in which the authors have systematically searched for, appraised, and summarised all of the medical literature for a specific topic. (qualitative review of evidence) Meta-analysis: A systematic review which uses quantitative methods to summarise the results

63. Best to least level of evidence a) prospective with history control, restrospective study, case report b) ??? c) metaanalysis, RCT, prospective d) systematic review, meta-analysis, RCT

ANSWER: D (consensus) Carcinoid tumours can be found in the lung but are a different entity from small cell carcinoma. SCLC causes a variety of paraneoplastic syndromes owing to the production of peptide hormones and anti- bodies. The most common of these is hyponatremia, usually as a result of an inappropriate production of antidiuretic hormone (SIADH). Cushing syndrome and hypercortisolism through ectopic production of adrenocorticotropic hormone are also commonly seen. Hypercalcemia is commonly associated with squamous cell lung cancers (NSCLC) because of elaboration of a parathyroid-like factor and not as a result of bone metastases. A rare neurologic paraneoplastic syndrome associated with small-cell lung tumors is the Lambert-Eaton (also called Eaton-Lambert) myasthenic syndrome caused by impaired release of acetylcholine from nerve terminals. This typically presents as proximal lower limb weakness and fatigability that may temporarily improve with exercise.Patients with LEMS are sensitive to the effects of both depolarizing and non-depolarizing muscle relaxants. The administration of 3,4-diaminopyridine should be continued until the time of surgery. The possibility of occult muscle weakness should be considered in any patient with a malignant tumor.

64. Small cell carcinoma associated with a) carcinoid b) hypercalcemia c) resistance to NMB d) SIADH

ANSWER: A (consensus) WHY: Although the median nerve can also be injured while inserting an antecubital IV, the paresthesia described is the sensory distribution of the lateral cutaneous nerve (forearm), not the median (sensory to hand). The musculocutaneous nerve continues as the lateral cutaneous nerve of the forearm at the antecubital fossa and courses along the lateral aspect of the forearm providing subsequent anterior and posterior branches. The median nerve crosses the elbow lying medially on the brachialis muscle and just medial to the brachial artery and vein (all of these are medial to the biceps brachii tendon). Distal to the antecubital fossa, the nerve gives off the anterior interosseous nerve and cutaneous sensory branches.

65. Following insertion of an antecubital IV, a patient develops pain on the lateral aspect of the forearm. Which nerve is most likely affected? a) Lateral cutaneous nerve of the forearm b) Median nerve c) Radial nerve d) Anterior Intraosseus nerve

Answer: B Fetal and neonatal effects: lethargy, hypotonia, respiratory depression and demineralization with prolonged use Pediatric Endocrinology Magnesium may cause mild peripheral arterial vasodilation. Magnesium ions cross the placenta readily and may lead to fetal and neonatal hypermagnesemia. There is poor correlation between magnesium concentrations in the umbilical cord blood and the incidence of low Apgar scores and depression of ventilation at birth, which are more likely due to fetal asphyxia and prematurity. Although some investigators have suggested that antenatal exposure to magnesium sulfate results in a higher risk for adverse neonatal outcomes, others have observed no association between umbilical cord blood magnesium concentration and the need for delivery room resuscitation when magnesium was administered for neuroprotection in anticipation of a preterm birth.

79. A parturient receives a bolus of MgSo4 4g and then a subsequent 2g/hr IV infusion. What effects will this have on the neonate? a) tachycardia b) hypotension c) seizures d) hyperglycemia

ANSWER: D (consensus) WHY: Miller says VO2 max < 15 is the best predictor and Barash says DLCO2 < 40 is the best predictor; the only parameter listed that everyone agrees is terrible is FEV1 < 40%. Traditionally, arterial blood gas data such as partial pressure of oxygen in the blood (PaO2) less than 60 mm Hg or partial pressure of carbon dioxide in the blood (PaCO2) greater than 45 mm Hg have been used as cutoff values for pulmonary resection. Cancer resections have now been successfully performed singly or even combined with volume reduction in patients who do not meet these criteria, although they remain useful as warning indicators of increased risk. Formal laboratory exercise testing is currently the gold standard for assessment of cardiopulmonary function, and the maximal oxygen consumption (VO2max) is the most useful predictor of postthoracotomy outcome. The risk of morbidity and mortality is increased if the preoperative VO2max is less than 15 mL/kg/min and is considered very high if it is less than 10 mL/kg/ min. (Miller) Patients with a predicted postoperative FEV1 value >40% are at reduced risk and those with predicted postoperative FEV1 <30% are at increased risk. Those patients who fall into the latter category are more likely to need postoperative ventilation. A predicted postoperative diffusing capacity for carbon monoxide <40% is associated with increased risk. Predicted postoperative diffusing capacity percent is the strongest single predictor of risk of complications and mortality after lung resection. (Barash)

66. 60 yo F for RLL pneumonectomy. What preOP investigations will predict a big risk of post OP pulmonary complications? a) PreOP PaO2 65 mmHg b) VO2 (max) 20mL/kg/min c) post op predicted DLCO2 50% d) post op predicted FEV1 20%

ANSWER: A (consensus) WHY: Although D is probably also true, our textbooks focus on the RECENT smoking effect of carboxyhemoglobin. Smoking leads to a prolonged period of tissue hypoxemia. Cessation of smoking 48 hours before surgery has been shown to decrease the percentage of carboxyhemoglobin, to shift the oxyhemoglobin dissociation curve to the right, and to increase oxygen availability. The rate-pressure product (RPP = heart rate × peak systolic arterial pressure) was considered an easily determined index of MV̇O2. Patients without ischemic heart disease who smoked shortly before operation have significantly more episodes of ST-segment depression than do nonsmokers, former smokers, or chronic smokers who do not smoke in the immediate pre- operative period. Smoking ... negatively affects coronary flow reserve; and causes vascular endothelial dysfunction, hypertension, and ischemia.

67. Effects of recent smoking include a) Decrease systemic O2 delivery b) Decreased RPP c) increase Coronary steal d) increase Platelet activating

ANSWER: EXCEPT B (consensus) Although not clinically proven, evidence suggests that second-generation SGAs, such as the pLMA and the sLMA, reduce the risk of aspiration of gastric contents. This property, along with the improved airway seal and higher leak pressures, have enabled SGA devices to be used in various applications where the cLMA is potentially unsuitable, such as in nonsupine positions (e.g., lateral, prone), in laparoscopic surgery and in patients who are obese. Predictors of failed LMA include advanced age, increased BMI, male, reduced thyromental distance, thick neck, poor dentition, smoking and surgical table rotation. Factors NOT associated with failed LMA include hyperreactive airway, sleep apnea, high Mallampati score, reduced mouth opening, reduced neck movement, limited jar protrusion, beard, novice user and non-supine.

68. Predictors for difficult LMA use a) poor dentition b) female c) increased BMI d) rotation of surgical table

ANSWER: D (consensus) or B(*except* consensus) HypoMg: in most symptomatic patients serum [Mg2+] is <1.2 mg/dL. Patients frequently complain of weakness, lethargy, muscle spasms, paresthesias, and depression. When severe, hypomagnesemia may induce seizures, confusion, and coma. Cardiovascular abnormalities include coronary artery spasm, cardiac failure, dysrhythmias, and hypotension. Symptoms that can accompany severe hyponatremia ([Na+] <120 mEq/L) include loss of appetite, nausea, vomiting, cramps, weakness, altered level of consciousness, coma, and seizures. Hypokalemia causes muscle weakness and, when severe, may even cause paralysis. Acute hypokalemia causes hyperpolarization of the cardiac cell and may lead to ventricular escape activity, reentrant phenomena, ectopic tachycardias, and delayed conduction. Characteristic electrocardiographic changes associated with hypokalemia include flat or inverted T waves, prominent U waves, and ST segment depression. Hypokalemia prolongs repolarization and leads to long QT(U) syndrome, predisposing to a torsades de pointes-type ventricular fibrillation. The hallmark of hypocalcemia is increased neuronal membrane irritability and tetany. Early symptoms include sensations of numbness and tingling involving fingers, toes, and the circumoral region. More severe symptoms include muscle cramps, laryngospasm, tetany and seizures. Delayed ventricular repolarization results in a prolonged QT interval on the ECG. Although prolongation of the QT interval may be a reliable sign of hypocalcemia in an individual patient, the ECG is relatively insensitive for the detection of hypocalcemia.

69. 80yo female with hip fracture appears slightly confused, and complains of weakness cramping and has prolonged QT on EKG. This is caused by a) hypoMg b) hypoNa c) hypoK d) hypoCa

Answer:D

71. According to ACLS 2010 for adults: a) Use chest compression depth of 2.5cm b) Once intubated, ventilate at a rate of 12-15 breaths per minute c) The adult compression to breath ratio is 15:2 d) In a witnessed cardiac arrest, start chest compressions before administering ventilations

ANSWER: B (consensus) WHY: Although tension pneumothorax is not a common complication to result from a stellate ganglion block, it is the only indication listed for immediate chest tube insertion. Diaphragmatic hemiparesis (A), tamponade (C) and gas trapping (D) do not necessitate a chest tube. Tension pneumothorax involving >50% of a hemithorax presents with dyspnea, tachycardia, cyanosis, agitation, diaphoresis, neck vein distention, tracheal deviation, and displacement of the maximal cardiac impulse to the contralateral side. Common complications of stellate ganglion blockade include temporary hoarseness and feeling of a lump in the throat (recurrent laryngeal nerve block), the unpleasant effects of Horner syndrome, hematoma, and neuralgia along chest wall and inner aspect of upper arm. Uncommon complications include brachial plexus injury (rarely), phrenic nerve block, pneumothorax, and osteitis of the transverse process. Severe complications include injection into the vertebral artery, producing immediate CNS effects with grand mal seizures, and intradural injection, with a slower onset of symptoms of spinal anesthesia.

72. Woman with obesity and asthma, has chronic left arm pain from Colles' fracture received a stellar ganglion block with 15mL of 0.25% bupivacaine. Thirty minutes later, she complained of dyspnea. What would be an indication for immediate chest tube insertion? a) elevated diaphragm on the left side on CXR b) right-sided deviation of trachea, 50% of left lung field lack of lung markings c) unable to sit up, SBP 60, HR 40 d) flattened diaphragm on inspiratory/expiratory film

ANSWER: B (consensus) Alveolar gas equation: PAO2 = PIO2 − PACO2/R PIO2 = FIO2 × (PB - PH2O) PIO2 = 0.21 × (760 - 47) = 150 mm Hg PAO2 = 150 - 80/0.8 PAO2 = 50 mm Hg

73. Patient with orthopaedic surgery at Halifax (sea level). He breathes ambient air and PaCO2 is 80. What is the PAO2 (repeat from 2008) a) 40 b) 50 c) 60 d) 70

ANSWER: C (consensus) WHY: Although epidural or paravertebral would provide the best pain control, his clopidogrel for recent DES would have to be stopped for minimum 7 days to safely perform a neuraxial/PVB technique and ought not be stopped in the first year after placement. Failing that, multimodal analgesia (for opioid-sparing effect, which will be beneficial for his OSA) with PCA is the safest choice. Effective pain relief, preferably with continuous thoracic epidural anesthesia or paravertebral or intercostal block, is central to management of rib fractures. (Barash ch. 52 p. 1509) The major concern with premature discontinuation of dual antiplatelet therapy, especially during critical time windows following bare metal stent implantation (30 days) or drug-eluting stent implantation (1 year), is the risk of precipitating catastrophic stent thrombosis, MI, or death. (Miller ch. 38 p. 1099) NSAIDs have proved effective in the treatment of postoperative pain. In addition, they are opioid-sparing and can significantly decrease the incidence of opioid-related side effects such as postoperative nausea and vomiting and sedation. (Barash ch. 56 p. 1623)

74. 50 year-old man suffered from a fall and fractured 7th and 8th ribs. He has a history of OSA on CPAP. He also has CAD with a drug-eluting stent placed 3 months ago. His medications include metoprolol, clopidrogrel, and atorvastatin. What is the BEST analgesia strategy? a) fentanyl PCA b) Epidural analgesia c) acetaminophen and morphine PCA d) paravertebral block

ANSWER: C (consensus) Currently validated techniques are 4-vessel cerebral angiogram or radionuclide cerebral blood flow imaging. EEG is no longer recommended. NDD can be confirmed by ancillary testing when minimum clinical criteria cannot be completed or confounding factors cannot be corrected. In cases of NDD for the purposes of postmortem donation, we recommend that any physician who has had any association with the proposed recipient that might influence the physician's judgment shall not take any part in the declaration of death. Deep unresponsive coma implies a lack of spontaneous movements as well as an absence of movement originating in the CNS, such as cranial nerve function, CNS-mediated motor response to pain in any distribution, seizures, decorticate and decerebrate responses. Spinal reflexes or motor responses confined to spinal distribution may persist. The legal time of death is marked by the first determination of death. Existing law states that for the purposes of postmortem donation, the fact of death shall be determined by 2 physicians. The physicians' determinations may be performed concurrently. If performed at different times, a full clinical examination including the apnea test must be performed, without any fixed examination interval, regardless of the primary etiology.

75. What is most consistent with the The Canadian Forum on " Severe brain injury to neurological determination of death" a) If mechanism is unknown, EEG is an acceptable method for determination of braindeath. b) Testing for brain death should not start until the patient's POA has been contacted first for organ donation. c) Movement can still occur in the presence of brain death due to spinal reflexes d) Legally, time of death is documented as the time of cardiovascular collapse and pulselessness.

ANSWER: D (consensus) Most of the intravenous induction agents have been used for ECT. Methohexital (1 to 1.5 mg/kg) is considered the "gold standard". Etomidate (0.15 to 0.3 mg/kg) is generally associated with longer seizure duration and is the preferred agent of some psychiatrists, despite a slightly longer recovery and associated myoclonus. Propofol (an anticonvulsant) is more effective at attenuating the acute hemodynamic responses than etomidate and in small doses (0.75 mg/kg) seizure duration is usually acceptable. Short-acting opioids, such as remifentanil, can be used to decrease the dose of induction agent and prolong seizure duration without reducing the depth of anesthesia.

76. During ECT, which agent is associated with the shortest seizure durations? a) Remifentanil b) Methohexital c) Etomidate d) Propofol

ANSWER: B (consensus) This is supported by a study showing that daily trials of unassisted ventilation ("T-piece trials") resulted in more rapid separation from ventilation than other more gradual approaches, in particular IMV "weaning." In addition, the so-called "weaning parameters" are inadequate predictors of the success or failure of withdrawal of ventilatory support and add little to routine management. Thus, the process of separation from mechanical ventilation is expedited when respiratory therapy-driven protocols are used that focus on daily assessment of the ability to breathe without assistance, assuming improvement of the inciting process, adequate oxygenation, and hemodynamic stability. Once the patient can breathe comfortably for 30 to 120 minutes without support, the trachea can be extubated, assuming that there are no other precluding factors such as airway abnormalities and coma.

77. A 50kg female has been ventilated for 10 days in the ICU on pressure control ventilation. She has a good gag, obeys commands. After a 1 min trial of CPAP of 5cmH20 with Fi02 100%, she is breathing at a resp rate of 20 bpm, and volumes of 350mL. What is the most appropriate next step? a) Sedation, and start pressure support ventilation b) Start spontaneous ventilation trial c) Sedation, and continue pressure control ventilation d) Consult for tracheostomy.

ANSWER: A (consensus) Closing capacity typically exceeds functional residual capacity in the mid-60s, and will eventually exceed the tidal volume at some later age. • Increased: chest wall stiffness, lung compliance, closing capacity (Exceeds FRC in mid 60s), VQ mismatch • Decreased: TLC, VC, IC, alveolar surface, hypercarbic drive (50%), hypoxic drive (> 50%) • Airway obstruction, sleep disordered breathing, decreased coughing force, aspiration

78. With respect to elderly physiology which statement is TRUE? a) Increased Closing capacity b) Decreased FRC c) Increased RLC d) Increased VC e) total lung volume increases

Consensus A Answer A loss of deep tendon reflexes although C could possible be true as well • Signs and symptoms o 4-5 mEq/L - lethargy, nausea, vomiting flushing o >6 mEq/L - loss of DTR, hypotension o >10 mEq/L - paralysis, apnea, heart block, and/or cardiac arrest CoE Chapter 18 pg 369 • 1. -2.4 mg/dL - normal range • 5-9 mg/dL - therapeutic range • 12mg/dL - patellar reflexes are lost • 15-20 mg/dL -respiratory arrest • >25 mg/dL - asystole Recall tx = calcium gluconate 1 g/10 minutes Chestnut pg 839 (note different units) Regarding C - possibly true • ECG (as for hyperkalemia) o Increase PR and QTc o Prolonged QRS o Peaked T waves and flattened p waves o Complete AV block and asystole

8. You are called to see a preeclamptic woman who is on magnesium perfusion. How would you know that there is an overdose of MgSO4? a) loss of deep tendon reflexes b) hyperreflexia and headahce c) peaked T wave d) fasiculations

ANSWER: C (consensus) Elevation HOB 90 papers Subglottic suction 60 papers WHY: This is the most recommended intervention based on cost-effectiveness and lack of potential adverse effects. Our textbooks, however, do not compare the relative risk reduction of these interventions head-to-head. The simplest and least expensive and yet very effective interventions are strict hand washing between patients and semirecumbent positioning of the patient (head height at 30 degrees or greater from horizontal). Acid-suppression therapies have been associated with an increased risk of VAP because they allow bacterial overgrowth in the stomach. Excessive use of gastric pH-altering medications for stress ulcer prophylaxis increases gastric pH and the risk for VAP. The use of sucralfate, an agent that does not increase gastric pH, may be preferable to H2-receptor antagonists or proton pump inhibitors. Somewhat more expensive interventions to reduce VAP that may be useful in certain patients include subglottic suctioning and oscillating beds. Intermittent subglottic suctioning using specially designed tracheal tubes has been shown to reduce the incidence of VAP in several small studies and may reduce the duration of mechanical ventilation and length of ICU stay. However, these tubes are more expensive than conventional tracheal tubes, and their cost-effectiveness has not been established, particularly if their use is instituted in all intubated patients. Fiberoptic bronchoscopy has been used to obtain protected specimen brush samples or bronchoalveolar lavage samples for quantitative culture. Patients were considered to have VAP if at least 103 colony-forming units (CFU)/mL of bacteria grew from the protected specimen brush sample or at least 104 CFU/mL of bacteria grew from the bronchoalveolar lavage fluid. The patients in the invasive management group had reduced mortality at day 14 (16.2% versus 25.8%, P = .022) and an increased number of antibiotic-free days (5.0 ± 5.1 versus 2.2 ± 3.5 days, P < 0.001). This study makes a compelling argument for a more definitive diagnosis of VAP before initiating antibiotic therapy.

80. To decrease VAP, which has the most evidence: a) Sucralfate / stomach pH b) NG suctioning / supraglottic suction? c) Semi-sitting position (HOB at 30 degrees) d) Bronchoalveolar lavage e) Prophylactic antibiotics

ANSWER: A The most common complication is hemorrhage (0.73%) because of the proximity of major vessels and the vascularity of certain tumors. Pneumothorax is the second most common complication (0.66%). Recurrent laryngeal nerve injury occurred in 0.34% of cases and was permanent in 50% of these cases. Other reported complications include acute tracheal collapse, tension pneumomediastinum, mediastinitis, hemothorax, and chylothorax.

81. Most common complication of mediastinoscpy: a) Hemorrhage b) RLN injury c) Pneumothorax d) Tracheal compression

ANSWER: D (consensus) WHY: As you can see in the formulae below, VO2 is, in fact, independent of DO2. The other options are false. Oxygen delivery to the tissues (DO2) is dependent on cardiac output (CO), regional blood flow, and oxygen-carrying capacity also known as the oxygen content (CaO2) of blood. ERO2 = extraction ratio of oxygen A reduction in SVO2 results from an increase in ERO2, from either an increase in VO2 or a decrease in DO2. A decrease in DO2 occurs in conditions such as hemorrhagic or hypovolemic shock. An increase in VO2 may occur in conditions such as stress, pain, shivering, sepsis, and thyrotoxicosis. Conversely, an increase in SVO2 indicates either an elevation in O2 supply (increased SaO2, Hb, or Qt) or a reduction in VO2, as occurs during hypothermia. Mixed venous hemoglobin saturation values reflect global, whole-body measurement. Therefore, regionally inadequate blood flow and tissue oxygen delivery (such as with limb or intestinal ischemia) can coexist with a normal or high mixed venous hemoglobin saturation. Oxygen saturation monitoring has been incorporated into central venous catheters. These catheters measure central venous saturation, measured in the superior vena cava. Normally, this saturation is around 70% versus 75% in the pulmonary artery. EXTRACTION is therefore 100% - 75% = 25% (except for cardiac muscle). To the extent that arterial hemoglobin saturation, oxygen consumption, and hemoglobin concentration remain stable, mixed venous hemoglobin saturation may be used as an indirect indicator of cardiac output. For example, when cardiac output falls, tissue oxygen extraction increases and the mixed venous blood will have a lower oxygen content and lower hemoglobin oxygen saturation.

82. With regards to the DO2 and VO2, which of the following is true a) A normal SVO2 is indicative of adequate tissue oxygenation b) A normal oxygen extraction is 70% c) CO is needed to calculate the oxygen extraction ratio d) In normal circumstances, the VO2 is independent of DO2

ANSWER: C (consensus) WHY: Although B and D arguably have merit, C is the best answer. The Bier block has multiple advantages, including ease of administration, rapidity of recovery, rapid onset, muscular relaxation, and controllable extent of anesthesia. It is an excellent technique for short (<90 minutes) open surgical procedures and for closed reductions of bony fractures. The tourniquet can be released safely after 25 minutes, but the patient should be closely observed for local anesthetic toxicity for several minutes after the tourniquet release. Slow injection of local anesthetic solutions at a distal site decreases the risk of toxicity. Cyclic deflation of the tourniquet at 10-second intervals increases the time to peak arterial lidocaine levels that can decrease potential toxicity. If 40 minutes has elapsed, the tourniquet can be deflated as a single maneuver. Between 20 and 40 minutes, the cuff can be deflated, reinflated immediately, and finally deflated after 1 minute to delay the sudden absorption of anesthetic into the systemic circulation, although this may not truly lower the eventual peak plasma local anesthetic levels achieved.

83. Regarding IV regional anesthesia, which is true? a) Must leave the cuff up for 40 mins b) For surgeries lasting 60mins c) Cyclic deflation increases safety d) Double cuff increases safety

ANSWER: B (consensus) WHY: Valve "lesion" implies pathology, not physiologic change due to pregnancy. Cardiac pathology in pregnancy: Historically, rheumatic mitral stenosis represented the most common cardiac condition encountered in pregnant women. This disease continues to be a major problem in the developing world and in certain immigrant populations in the United States. In the industrialized countries, congenital heart disease has become the most common cardiac condition complicating pregnancy. Normal maternal physiology: First heart sound is accentuated with splitting. Second heart sound is unchanged. S4 may be heard. Grade II systolic murmur corresponds to tricuspid regurgitation. ECG shows decrease PR and uncorrected QT. QRS shifts to right early pregnancy and then left in third trimester. Right-sided ST may be depressed and T wave flattened. Echocardiography demonstrates hypertrophy, which is eccentric. Mitral, tricuspid and pulmonic annulus dilates. 94% shows tricuspid and pulmonic regurgitation and 27% shows mitral regurgitation.

84. Most common valve lesion in pregnancy? a) AS b) MS c) MR d) AR

ANSWER: A (consensus) Chronic hypoxemia affects all major organ systems. Compensatory mechanisms that attempt to provide adequate systemic oxygen delivery in the presence of chronic hypoxemia include polycythemia, increases in blood volume, alterations in oxygen uptake and delivery, and neovascularization. Despite the favorable effects of the adaptive responses, these alterations may be detrimental. Polycythemia, the most significant compensatory response, is associated with increases in blood viscosity and red cell sludging

85. With respect to chronic hypoxemia in peds, which is true: a) polycythemia b) neovascularization c) altered oxygen transport d) can't remember

Answer: A - It is difficult to determine the actual FiO2 Miller Ch. 55 Pg. 1678; Ch. 85 Pg. 2533 A - It is difficult to determine the actual administered FiO2 - TRUE: Jet ventilation works through the venturi effect: each high pressured puff of oxygen also entrains room air, which dilutes the FiO2. The actual inspired Oxygen content is unknown, and the tidal volume also cannot be measured. B - It is safer to apply the flow of oxygen perpendicularly to avoid barotrauma - FALSE: Barotrauma is avoided by ensuring there is no upper airway obstruction thereby allowing egress of air from an unobstructed glottis. Allowing for an adequate expiratory time (1 second on, 3 seconds off), and the use of no more than the minimum pressure required to adequately oxygenate and ventilate the patient are other measures taken to avoid barotrauma. Applying oxygen perpendicularly (to the trachea?) is not mentioned, and conceptually could lead to subcutaneous/mediastinal emphysema or perforation of the trachea/esophagus. C - Need to give a minimum of 30 psi - FALSE: The minimum pressure required to drive jet ventilation is 15psi. Jet ventilators usually contain an apparatus to lower the oxygen supply from the hospital line (usually at 55psi). Anywhere from 20 to 50 psi is typically used.

86. Regarding Jet ventilation a) It is difficult to determine the actual administered FI02 b) It is safer to apply the flow of oxygen perpendicularly to avoid barotrauma c) Need to give a minimum of 30 psi d) ?

Answer: A - OSA? or C - TMD < 6cm? Miller Ch. 38 Pg. 1091-1092 Barash Pg. 768 Table 27-7 Patients with the following characteristics have potential difficulty with ventilation through a bag-mask apparatus: (These 5 criteria are listed by both Barash and Miller ¥ Age >55 (Barash; Miller says 55+) (f) ¥ BMI >26 (includes e) ¥ Lack of teeth ¥ Beard (d) ¥ History of Snoring (b) Patients with the following characteristics may pose a challenge to ventilate by any means: ¥ OSA (a) ¥ snoring (b) ¥ Obesity ¥ Neck circumference >17 inches in men, >16 inches in women ¥ Thyromental distance <7cm with head in maximum extension (includes c) ¥ Higher Mallampati scores ¥ Large tongue ¥ Inability to protrude the mandible or lower teeth in front of the upper teeth ¥ Facial/neck deformities from other surgeries ¥ Previous head and neck radiation ¥ Head and neck trauma ¥ Congenital abnormalities of the head and neck ¥ Rheumatoid arthritis ¥ Down Syndrome ¥ Scleroderma ¥ Cervical spine disease or previous cervical spine surgery Unsure how to answer this. If you stick with factors strictly relating to difficulty bag mask ventilating the patient, OSA and TMD<6 are not on the list (but they do make the list of difficult to ventilate in ANY way). I'm not sure why there are so many options for a single-year, non-repeated question, but conceivably there is more than one wrong answer if the original question(s) had only 4 options. Age, BMI, Teeth, Beard, and snoring are predictors; the others not so much.

87. All are predictors of difficult BMV except: a) OSA b) History of Snoring c) TMD < 6 cm d) Beard e) BMI > 28 kg/m2 f) Age > 55 years

Answer: All EXCEPT B - Umbilical artery Coexisting Ch. 26 Pg. 571 Miller/Barash/Chestnut all talk about AFE from a slightly different angle. It appears that AFE is no longer thought to be a purely embolic phenomenon, but more of an immune mediated one leading to right heart failure and cardiovascular collapse. Meconium, fetal squames/lanugo hair, and amniotic fluid entering the maternal circulation appear to be a fairly common occurrence, but do not routinely trigger AFE, and even when injected intentionally into animal models, do not always precipitate AFE. The traditional thought of precipitous and hard labour predisposing to AFE is questioned in Chestnut, where they give the very reasonable explanation that tachysystole shouldn't precipitate AFE, as the strong contractions lead to minimal uterine blood flow, and therefore couldn't cause amniotic fluid to enter the circulation (because there isn't any blood getting to the uterus during the strong contractions). Instead the tachysystole and fetal distress are thought to be symptoms of an impending AFE, not the cause. Coexisting discusses the three most common sites for entry of amniotic fluid into the maternal circulation: they are the endocervical veins, the placenta, and a site of uterine trauma. Chestnut mentions uterine trauma as a potential precipitant to AFE, but outside of that, Chestnut/Barash/Miller do not discuss potential sites of entry of amniotic fluid into the maternal circulation. Also the obvious: the umbilical artery is attached to the fetus, and not part of the maternal circulation; it interfaces with the placenta, which would then interface with the maternal circulation

89. Mechanism of AFE a) Endocervical veins b) Umbilical artery c) Placenta d) Site of uterine trauma

Consensus A - 10 schools C - 4 schools Answer A - low doses anti-platelet effect by thromboxane • This may be a controversial choice because ASA does not directly affect thromboxane rather prevents formation of A2 - semantics o Aspirin irreversibly acetylates cyclooxygenase and thereby prevents formation of thromboxane A2 (P&P 1138) Answer C- high and chronic doses never hand an effect on PT/PTT • Aspirin has no effect on PT/PTT (Barash and Miller) • However, the wording of "never" is highly suspicious and I could not find anything about high/chronic doses Regarding B low doses cannot augment bleeding time in healthy patient - false, low dose ASA will increase bleeding time, however high dose ASA may normalize this. Regarding D reversible inhibition of platelet aggregation - false • Despite rapid clearance from the body, the effects of aspirin on platelets are irreversible and last for the life of the platelet, 7-10days (P&P pdf 1138)

9. Concerning ASA, which is true? a) Low doses anti-platelet effect by thromboxane b) low doses cannot augment bleeding time in healthy patient c) High and chronic doses never have an effect on PT/PTT. d) Reversible inhibition of platelet aggregation

Answer: A Coexisting Ch. 25 pg. 545 A - Propranolol is the best choice for treatment - FALSE: Nitroglycerine is used to treat the myocardial ischemia related to cociane intoxication. Beta blockers may potentially worsen the coronary artery vasospasm associated with cocaine use. Alpha blockade can be employed to treat coronary vasospasm. If seizures develop, benzodiazepines are effective, in addition to active cooling if hyperthermia coexists with seizures. B - CCBs can be used to treat tachycardia and ischemia : nitroglycerine and alpha blockade are the drugs of choice. Some evidence for CCV and ischemia non responsive to nitro C - Effects are due to inhibition of reuptake both peripheral and central catecholamines - TRUE: Cociane works by preventing the presynaptic reuptake of dopamine and norepinephrine, leading to an increased postsynaptic concentration of both of these neurotransmitters. D - Cocaine toxicity causes negative myocardial function : Cocaine overdose leads to an overwhelming sympathetic stimulation of the cardiovascular system. Ultimately, coronary artery vasospasm and myocardial infarction would ultimately lead to negative myocardial function, but that is more a consequence of the excessive sympathetic stimulation, not a direct effect

90. Cocaine toxicity. All EXCEPT a) Propranolol is the best choice for treatment b) CCBs can be used to treat tachycardia and ischemia c) Effects are due to inhibition of reuptake both peripheral and central catecholamines d) Cocaine toxicity cause negative myocardial function

Answer: B - The patient is able to formulate idea that the physician agrees with Miller Ch. 10 Pg. 236 Answers A, C and D are the three criteria/concerns that one should address when assessing competence: 1) can the patient receive and understand the information relevant to the decision? 2) can the patient understand potential consequences of the decision, including potential risks and benefits, even in a simplified way? 3) can the patient express a decision and communicate his or her values regarding the medical advice being given? B - The patient is able to formulate idea that the physician agrees with - FALSE: "Functional capacity for decision making must be judged separately from the perceived quality of the decision itself. Patients have the right to make 'bad' decisions if they are competent and have appropriate information"

91. Patient competence and decision making. All true EXCEPT a) The patient is able to communicate ideas about their decision b) The patient is able to formulate idea that the physician agrees with c) The patient is able to receive information d) The patient is able to understand consequences of their decision

Answer: D - Pulmonary stenosis Coexisting Ch. 2 Pg. 38, 42, 44-45 Ch. 21 Pg. 444 MVP - TRUE: MVP can be associated with Marfan's syndrome, rheumatic carditis, myocarditis, thyrotoxicosis, and systemic lupus erythematosus. Aortic Insufficiency - TRUE: Common causes of lea et abnormalities are infective endocarditis, rheumatic fever, bicuspid aortic valve, and the use of anorexigenic drugs. Abnormalities of the aortic root causing aortic regurgitation include idiopathic aortic root dilation, hypertension-induced aortoannular ectasia, aortic dissection, syphilitic aortitis, Marfan's syndrome, Ehlers- Danlos syndrome, rheumatoid arthritis, ankylosing spondylitis, and psoriatic arthritis. Aortic aneurysm - TRUE: Marfans syndrome is associated with defective connective tissue in the aorta and heart valves, which can lead to aortic dilation, dissection, or rupture and to prolapse of cardiac valves, especially the mitral valve. Mitral regurgitation resulting from mitral valve prolapse is a common abnormality. Pulmonary stenosis - FALSE: is usually acquired congenitally, either as an isolated disorder, or as a part of a complex congenital disorder. Acquired forms of pulmonary stenosis can be due to rheumatic fever, carcinoid syndrome, infective endocarditis, or previous surgery or other interventions.

92. Which cardiac lesion is not associated with Marfans syndrome? a) MVP b) Aortic insufficiency c) Aortic aneurysm d) Pulmonary stenosis

Answer: C - Diffuse homogenous alveolar collapse Bilateral infiltrate on CXR - TRUE: Diffuse infiltrates consistent with non-cardiogenic pulmonary edema is a diagnostic criteria used for ARDS. Hypoxemia due to alveolar edema - TRUE: Acute refractory hypoxemia in the presence of diffuse infiltrates consistent with pulmonary edema, and a PaO2/FiO2 ratio typically less than 200 (or in less severe forms less than 300) is consistent with ARDS. A PCWP < 18mmHg is also used to rule out a cardiac cause of the pulmonary edema. Diffuse homogenous alveolar collapse - FALSE: Not a part of ARDS. Option may be meant to confuse you when you think of the use of PEEP in ARDS (falsely implying that alveolar collapse is inherent to the disease with PEEP used to counteract it). PEEP is applied to reduce the "volutrauma" associated with repetitive opening and closing of alveoli, as well as helping to maintain lung volumes, reduce R->L intrapulmonary shunting, and V/Q mismatch. Decreased lung compliance - TRUE: In the acute phase of ARDS, protein rich edema fluid enters the alveoli due to increased membrane permeability in the alveolar capillaries. Neutrophil mediated lung injury is seen, and pro inflammatory cytokines are involved. Once this acute phase has resolved, a chronic 'fibrosing alveolitis' can develop, with persistent arterial hypoxemia, and decreased pulmonary compliance.

93. Regarding ARDS, which is not correct? a) Bilateral infiltrate on CXR b) Hypoxemia due to alveolar edema c) Diffuse homogenous alveolar collapse d) Decreased lung compliance

Answer: This question does not make sense. The only true statement is that propofol decreases the CMR (Option A). Miller Ch. 17 Pg. 398 The remainder of the options are false statements. There is no discussion of the mechanism through which propofol leads to EEG changes, only a discussion of the pattern of changes seen on EEG: First an activation, followed by dose related depression. As the patient looses consciousness, frontal spindles are seen, then are replaced by polymorphic 1- to 3- Hz activity. Further increases in propofol dose leads to burst suppression, and ultimately EEG silence. Miller Ch. 49 Pg. 1512 A - Decreases CMR - TRUE: Propofol decreases both Cerebral Metabolic Rate and Cerebral Blood Flow: surgical levels of propofol decreased CBF by 53%, and CMR by 79% in healthy volunteers. There is no discussion suggesting that this is or is not responsible for EEG changes. This is the only true statement regarding the effects of propofol on the brain, so the closest to answering this non-question. B - Decreased CPP - FALSE?: Propofol, when compared to an anesthetic based on an isoflurane/fentanyl or sevoflurane/fentanyl regime reduced subdural pressure in patients with intracranial tumours, suggesting that propofol decreases cerebral blood flow, cerebral blood volume and ICP. Considering that CPP = MAP - ICP (and propofol would decrease both MAP and ICP), it is unclear what the net effect would be. Considering however, that inhalational anesthetics would also decrease your MAP comparably to propofol, I'm inclined to think that CPP is maintained, or even improved with a propofol based anesthetic, making this statement false. C - Altered response to CO2 - FALSE: CO2 responsiveness is preserved in humans administered propofol, even at levels resulting in burst suppression. CO2 reactivity is a robust mechanism and is preserved under all anesthetic conditions. (Barash D - Altered cerebral autoregulation - FALSE: Cerebral autoregulation is preserved in humans given propofol, even at doses resulting in burst suppression of the EEG.

94. Which of the following is not responsible for propofol's effect on the EEG (something like that)? a) Decreases CMR b) Decreased CPP c) Altered response to CO2 d) Altered cerebral autoregulation

Answer: D - Best anesthetic choice is a RSI Chestnut Ch. 50 Pg. 1144, 1149, Ch. 23 Pg. 478 A - Increased incidence of Shoulder Dystocia - TRUE: Not much more to say. B - Elevated risk for operative delivery - TRUE: obesity increases the risk for both operative vaginal delivery, and cesarean section (I'm presuming Chestnut refers to operative vaginal delivery as the use of instruments for extraction, or an episiotomy?). C - Continuous spinal catheter is an acceptable anesthetic technique for C- Section - TRUE: While microcatheters are no longer available, and implicated in the development of cauda equina syndrome (though this is postulated to be related to maldistribution of local anesthetic, and not necessarily the catheters themselves), continuous spinal catheters are an option when unintended dural puncture has occurred, or in patients in whom placement of an epidural catheter is difficult (ie. morbid obesity, abnormal vertebral anatomy such as kyphoscoliosis, or in patients with severe cardiac disease who require careful titration of analgesia). [I don't see how the last of those three fits into the category of difficult epidural insertion, but this is what Chestnut has written]. The situation of a morbidly obese patient and an emergency cesarean section is where a continuous spinal catheter is advocated, as insertion of a single shot spinal using a spinal (= small) needle may both be technically challenging, and temporally insufficient for the duration of surgery. To avoid the potential difficulty of insertion, and the even more difficult situation of having to induce a general anesthetic during an operative delivery, a continuous spinal technique (using a touhey needle to insert) , or a CSE approach is advocated. D - Best anesthetic choice is a Rapid Sequence Induction - FALSE: Neuraxial anesthesia is the best choice for obese parturients.

95. Obese woman for pregnancy. All of the following statements are true a except for one. a) Increased incidence of Shoulder Dystocia b) Elevated risk for operative delivery c) Continuous spinal catheter is an acceptable anesthetic technique for C-Section. d) Best anesthetic choice is a Rapid Sequence Induction

Answer: A - Glycopyrrolate Chestnut Ch. 4 Pg. 66-68 I recognize that the F/M ratio for IM glycopyrrolate is higher than Nitroglycerin, however the wording of the paragraphs discussing each drug seem more pessimistic about the placental transfer of glycopyrrolate, and of the clinically observable effects on the fetus. Note that IV glycopyrrolate is not given an F/M ratio, just an explanation that there is no detectible response in the fetus to its administration. Glycopyrrolate - No: Placental transfer of anticholinergic agents directly correlates with their ability to cross the blood-brain barrier. Glycopyrrolate is poorly transferred across the placenta, with IM administration resulting in a Fetal/Maternal ratio of only 0.22, and an IV administration resulting in no detectable fetal hemodynamic response. Nitroglycerin - Yes?: Nitroglycerin crosses the placenta to a limited extent, with an Fetal/Maternal ratio of 0.18, and results in minimal changes in fetal umbilical blood flow, blood pressure, heart rate, and blood gas measurements in gravid ewes. Dinitrate metabolites are found in both maternal and fetal blood, indicating that the placenta can metabolize nitroglycerin. Midazolam - Yes: has a Fetal/Maternal ratio of 0.76 at 20 minutes, and 200 minutes after administration the Fetal/Maternal concentration ratio decreases to 0.3 Propofol - Yes: has a mean Fetal/Maternal ratio between 0.65 and 0.85.

96. All of the following drugs cross the placenta except for one, name the exception. a) Glycopyrrolate b) Nitroglycerin c) Midazolam d) Propofol

Answer: C - Dry absorber par decomposition Barash Pg. 451-452 FA/FI is the fractional concentration of alveolar anesthetic divided by the fractional concentration of Inhaled anesthetic. Collapsed reservoir bag - FALSE: This will decrease the circuit capacity. The circuit is filled initially with non-anesthetic containing gas, which dilutes the anesthetic entering from the fresh gas outlet. The larger the circuit capacity is, the more of a dilution/decrease in the inspired anesthetic concentration there will be. In this situation, collapsing the reservoir bag would result in a more concentrated inspired anesthetic fraction, or Fi, which would lead to a more rapid rise in FA/FI. Don't think of it as a ratio of two independent factors: The alveolar anesthetic fraction is dependent on the inspired fraction, so even though in this situation we are doing something to increase the denominator, the result is an increase in the ratio, because a higher Fi will also lead to a higher FA. Short breathing circuit tubing - FALSE: See explanation above. A shorter circuit means less of the fresh gas flow is diluted by non-anesthetic gas containing air. A shorter circuit would increase Fi, which would increase FA/FI Dry absorber par decomposition - TRUE: What this option is trying to get at (in French) is that a dry CO2 absorber can adsorb and decompose inhaled anesthetics, which theoretically lead to a delayed rise of Fi, and by extension a delayed FA/FI. Clinically, however this does not affect the rate of rise of Fi to a significant extent compared with other factors. Increased minute ventilation - FALSE: an increased minute ventilation would speed up the equilibration between FA/FI, and increase the fraction of alveolar/inspired anesthetic agent. A nice table that includes all of the factors that increase/decrease FA/Fi:

98. Which of the following decrease FA/FI: a) Collapsed reservoir bag b) Short breathing circuit tubing c) Dry absorber Par décomposition d) increased minute ventilation

Answer: A - Lower blood: gas partition coefficient Miller Ch. 26 Pg. 641 Barash Pg. 449 Lange Pg. 156, 162 Lower blood gas partition coefficient - TRUE: Blood gas partition coefficient for Desflurane is 0.45, for Isoflurane is 1.4, and for Sevoflurane is 0.65. The lower the blood:gas partition coefficient, the less avidly the gas is taken up by the blood, which leads to a higher rise in the alveolar concentration (which is conceptually though to represent brain anesthetic concentration), resulting in a faster induction. A higher blood:gas partition coefficient represents a greater solubility of the anesthetic gas, resulting in greater uptake by the pulmonary circulation, and a slower rise in the alveolar partial pressure, resulting in a prolonged induction. Anesthetic blood:gas and tissue:blood coefficients are important factors in uptake and distribution of inhaled anesthetics, as they move from the alveoli to the blood, then from the blood to various tissues. High vapour pressure - FALSE: The vapour pressure is the maximal partial pressure of a volatile compound. The partial pressure is the portion of the total pressure that the gas in question applies to a gas mixture. The vapour pressure of Desflurane is 669, of Isoflurane is 238, and of Sevoflurane is 157. While Desflurane's vapour pressure is higher than Sevo/Iso, this does not have a bearing on speed of induction, but instead on how the gas is delivered (requiring a special vaporizer compared with Sevo/Iso). A MAC value relatively higher - FALSE: MAC of desflurane is 6%, of isoflurane is 1.28, and of sevoflurane is 2%. The Minimal Alveolar Concentration is the alveolar concentration that prevents movement in 50% of patients in response to a standard stimulus. It is meant to mirror brain partial pressure, and to allow for comparison of potency between agents. Doesn't reflect speed of induction, nor the solubility of the gas. A low oil:Blood partition coefficient - FALSE: Not sure that this partition coefficient exists. The closest I could find to an oil:blood partition coefficient was a fat:blood partition coefficient, which for desflurane is 27, for isoflurane is 45, and for sevoflurane is 48. I'm unsure of the usefulness of this coefficient, other than to indicate that the blood solubility of the gas increases more following postprandial lipemia (you eat, there's more fat in your blood, and the higher this number, the more soluble the gas becomes as a result), and decreases with anemia (don't have an explanation here). If my understanding of this partition coefficient is correct, having a lower fat:blood coefficient would mean that desflurane solubility is less impacted by the amount of fat in your blood.

99. What explains the faster induction of general anesthesia with desflurane? a) lower blood:gas partition coefficient b) high vapor pressure c) a MAC value relatively higher d) a low oil:blood partition coefficient


Conjuntos de estudio relacionados

Billing and coding exam style questions

View Set

1 - Escoger (Chapter 3)Audio You will hear some questions. Select the correct answers below based on the family tree.

View Set

Сучасні технології навчання

View Set

U.S. History Chapters 11-15 Multiple Choice

View Set

The Great Depression: Herbert Hoover Part 3

View Set

Food Science Final Test Questions

View Set

Pharmacology II Prep U Chapter 38: Agents to Control Blood Glucose Levels

View Set

HIST 202: Part 1; Lesson 6 "What is Populism?"; "When We Hear Populism."

View Set